July 2013
MEE Questions
and Analyses
National Conference of Bar Examiners
302 South Bedford Street | Madison, WI 53703-3622
Phone: 608-280-8550 | Fax: 608-280-8552 | TDD: 608-661-1275
www.ncbex.org
e-mail: contact@ncbex.org
Copyright © 2013 by the National Conference of Bar Examiners.
All rights reserved.
Contents
Preface....................................................................................................................................................... ii
Description of the MEE
............................................................................................................................ii
I
nstructions
............................................................................................................................................... iii
Ju
ly 2013 Questions
Federal Civil Procedure Question* .
...................................................................................................3
Ag
ency and Torts Question* .............................................................................................................4
Family Law Question
........................................................................................................................5
Ev
idence Question
* ...........................................................................................................................6
Negotiable Instruments Question
......................................................................................................8
Corpora
tions Question
* .....................................................................................................................9
Contracts Question*......................................................................................................................... 10
Real Property Question
.................................................................................................................... 11
Deced
ents’ Estates Question
* .......................................................................................................... 12
July 2013 Analyses
Federal Civil Procedure Analysis
.................................................................................................... 15
Agency and Torts Analysis.............................................................................................................. 20
Family Law Analysis
....................................................................................................................... 23
Evidence Analysis
........................................................................................................................... 27
Negotiable Instruments Analysis
..................................................................................................... 31
Corporations Analysis
..................................................................................................................... 34
Contracts Analysis ........................................................................................................................... 38
Real Property Analysis
.................................................................................................................... 42
Decedents’ Estates Analysis
............................................................................................................ 46
*Used as one of the six questions on the July 2013 Uniform Bar Examination in Alabama, Arizona, Colorado, Idaho, Missouri, Montana,
Nebraska, North Dakota, Utah, Washington, and Wyoming.
i
Preface
The Multistate Essay Examination (MEE) is developed by the National Conference of Bar
Examiners (NCBE). This publication includes the questions and analyses from the July 2013
MEE. Each test includes nine 30-minute questions; user jurisdictions may elect which of the nine
questions they wish to use. (Jurisdictions that administer the Uniform Bar Examination [UBE]
use a common set of six MEE questions as part of their bar examinations.) In the actual test, the
questions are simply numbered rather than being identified by area of law. The instructions for
the test appear on page iii. For more information, see the MEE Information Booklet, available on
the NCBE website at www.ncbex.org.
The model analyses for the MEE are illustrative of the discussions that might appear in excellent
answers to the questions. They are provided to the user jurisdictions to assist graders in grading
the examination. They address all the legal and factual issues the drafters intended to raise in the
questions.
The subjects covered by each question are listed on the first page of its accompanying analysis,
followed by roman numerals that refer to the MEE subject matter outline for that subject. For
example, the Federal Civil Procedure question on the July 2013 MEE tested the following areas
from the Federal Civil Procedure outline: I.A.1., B. & E., Jurisdiction and venue—Subject matter
jurisdictionFederal courts, Jurisdiction over parties, and Venue, forum non conveniens, and
transfer. Subject matter outlines are included in the MEE Information Booklet.
Description of the MEE
The MEE consists of nine 30-minute essay questions, any of which a jurisdiction may select to
include as part of its bar examination. (UBE jurisdictions use a common set of six MEE
questions as part of their bar examinations.) It is administered by participating jurisdictions on
the Tuesday before the last Wednesday in February and July of each year. The areas of law that
may be covered by the questions on any MEE are Business Associations (Agency and
Partnership; Corporations and Limited Liability Companies), Conflict of Laws, Constitutional
Law, Contracts, Criminal Law and Procedure, Evidence, Family Law, Federal Civil Procedure,
Real Property, Torts, Trusts and Estates (Decedents’ Estates; Trusts and Future Interests), and
Uniform Commercial Code (Negotiable Instruments and Bank Deposits and Collections; Secured
Transactions). Some questions may include issues in more than one area of law. The particular
areas covered vary from exam to exam.
The purpose of the MEE is to test the examinee’s ability to (1) identify legal issues raised by a
hypothetical factual situation; (2) separate material which is relevant from that which is not; (3)
present a reasoned analysis of the relevant issues in a clear, concise, and well-organized
composition; and (4) demonstrate an understanding of the fundamental legal principles relevant
to the probable solution of the issues raised by the factual situation. The primary distinction
between the MEE and the Multistate Bar Examination (MBE) is that the MEE requires the
examinee to demonstrate an ability to communicate effectively in writing.
ii
Instructions
The back cover of each test booklet contains the following instructions:
You will be instructed when to begin and when to stop this test. Do not break the seal on this
booklet until you are told to begin.
You may answer the questions in any order you wish. Do not answer more than one question
in each answer booklet. If you make a mistake or wish to revise your answer, simply draw a
line through the material you wish to delete.
If you are using a laptop computer to answer the questions, your jurisdiction will provide you
with specific instructions.
Read each fact situation very carefully and do not assume facts that are not given in the
question. Do not assume that each question covers only a single area of the law; some of the
questions may cover more than one of the areas you are responsible for knowing.
Demonstrate your ability to reason and analyze. Each of your answers should show an
understanding of the facts, a recognition of the issues included, a knowledge of the applicable
principles of law, and the reasoning by which you arrive at your conclusion. The value of
your answer depends not as much upon your conclusions as upon the presence and quality of
the elements mentioned above.
Clarity and conciseness are important, but make your answer complete. Do not volunteer
irrelevant or immaterial information.
Answer all questions according to generally accepted fundamental legal principles unless
your jurisdiction has instructed you to answer according to local case or statutory law. (UBE
instructions: Answer all questions according to generally accepted fundamental legal
principles rather than local case or statutory law.)
iii
July 2013 MEE
QUESTIONS
Federal Civil Procedure
Agency and Torts
Family L aw
Evidence
Negotiable Instruments
Corporations
Contracts
Real Property
Decedents’ Estates
FEDERAL CIVIL PROCEDURE QUESTION _______________
A woman was born and raised in the largest city (“the city”) of State A, where she also attended
college.
Three years ago, the woman purchased a 300-acre farm and a farmhouse in neighboring State B,
50 miles from the city. She moved many of her personal belongings to the State B farmhouse,
registered her car in State B, and acquired a State B driver’s license. She now spends seven
months of the year in State B, working her farm and living in the farmhouse. She pays income
taxes in State B, but not in State A, and lists State B as her residence on her federal income tax
returns.
However, the woman has not completely cut her ties with State A. She still lives in the city for
five months each year in a condominium that she owns. She still refers to the city as “home” and
maintains an active social life there. When she is living on the farm, she receives frequent
weekend visits from her city friends and occasionally spends the weekend in the city at her
condominium. She is a member of a health club and a church in the city and obtains all her
medical and dental care there. She is also registered to vote and votes in State A.
A food product distributor sells food items to grocery stores throughout a five-state region that
includes States A and B. The distributor is a State C corporation. Its corporate headquarters are
in State B, where its top corporate officers, including its chief executive officer (CEO), have
their offices and staff. The distributor’s food processing, warehousing, and distribution facilities
are all located in State A.
Three years ago, the woman and the distributor entered into a 10-year written contract providing
that the woman would sell all the produce grown on her farm each year to the distributor. The
contract was negotiated and signed by the parties at the distributor’s corporate headquarters in
State B.
The woman and the distributor performed the contract for two years, earning her $80,000 per
year. Recently, the distributor decided that the woman’s prices were too high. At a meeting at its
corporate headquarters, the distributor’s CEO asked the woman to drop her prices. When she
refused, the CEO informed her that the distributor would no longer buy produce from her and
that it was terminating the contract.
The woman has sued the distributor for anticipatory breach of contract. She seeks $400,000 in
damages. She has filed suit in the United States District Court for the District of State A,
invoking the court’s diversity jurisdiction.
State A’s long-arm statute provides that “a court of this State may exercise personal jurisdiction
over parties to the fullest extent permitted by the due process clause of the Fourteenth
Amendment to the United States Constitution.”
The distributor has moved to dismiss the woman’s action for lack of subject-matter jurisdiction
and for improper venue.
1. Should the court grant the motion to dismiss for lack of subject-matter jurisdiction?
Explain.
2. Should the court grant the motion to dismiss for improper venue? Explain.
3
AGENCY AND TORTS QUESTION _______________
After a dump truck unloaded gravel at a road construction job site, the trucker negligently drove
away with the truck bed still in a raised position. The raised truck bed hit an overhead cable,
causing it to fall across the highway.
The telephone company that owned the fallen cable sent one of its employees to the scene in a
company vehicle. The employee’s responsibilities were expressly limited to responding to cable-
damage calls, assessing damage, and reporting back to the telephone company so that a repair
unit could be dispatched.
The foreman of the road construction job site asked the telephone company employee if the
foreman’s crew could lift the cable off the highway. Fearful that the cable might be damaged by
traffic, the telephone company employee said, “Go ahead, pick it up. Just don’t damage the
cable.” The foreman then directed his crew to stretch the cable over the highway so that traffic
could pass underneath.
Shortly thereafter, a bus passing under the telephone cable hit the cable and dislodged it, causing
the cable to strike an oncoming car. The driver lost control of the car and hit a truck carrying
asphalt to the road construction site. As a result of the collision, hot asphalt spilled and severely
burned the foreman.
The foreman is now threatening to sue the telephone company on the ground that it is responsible
for its employee’s negligence in authorizing the road construction crew to stretch the cable
across the highway. The telephone company argues that, even assuming that its employee was
negligent, the telephone company is not liable because:
1. the telephone company employee’s acts were outside the scope of his employment and
thus cannot be attributed to the telephone company;
2. there is no other agency theory under which the foreman could hold the telephone
company liable for its employee’s acts; and
3. the telephone company employee’s acts were not the proximate cause of the foreman’s
injuries.
Assess each of the telephone company’s responses.
4
FAMILY LAW QUESTION
Seven years ago, a married couple had a daughter.
Recently, the mother joined a small religious group. The group’s members are required to
contribute at least half their earnings to the group, to forgo all conventional medical treatments,
and to refrain from all “frivolous” activities, including athletic competitions and sports. The
mother has decided to adhere to all of the group’s rules.
Accordingly, the mother has told the father that she has given half of her last two paychecks to
the group and that she plans to continue this practice. The father objects to this plan and has
accurately told the mother that “we can’t pay all the bills without your salary.”
The mother has also said that she wants to stop giving their daughter her prescribed asthma
medications. The father opposes this because the daughter has severe asthma, and the daughter’s
physician has said that regular medication use is the only way to prevent asthma attacks, which
can be life-threatening. The mother also wants to stop the daughter’s figure-skating lessons. The
father opposes this plan, too, because their daughter loves skating. Because the father works
about 60 hours per week outside the home and the mother works only 20 hours, the father is
afraid that the mother will do what she wants despite his opposition.
The mother, father, and daughter continue to live together. They do not live in a community
property jurisdiction.
1. Can the father or the state child welfare agency obtain an order
(a) enjoining the mother from making contributions from her future
paychecks to the religious group? Explain.
(b) requiring the mother to take the daughter to skating lessons? Explain.
(c) requiring the mother to cooperate in giving the daughter her prescribed
asthma medications? Explain.
2. If the father were to file a divorce action against the mother, could a court award custody
of the daughter to him based on the mother’s decision to follow the religious group’s
rules? Explain.
5
EVIDENCE QUESTION
The city police department received a 911 call regarding a domestic violence incident. The caller
said that she was staying with her sister and her sister’s boyfriend. The caller said that she had
called the police because her sister’s boyfriend was becoming violent. The police department
records all 911 calls. The relevant portions of the 911 recording are as follows:
Caller: My sister’s boyfriend is out of control right now. He just threw a broken beer
bottle at my sister. It hit her on the arm. Now he’s holding a chair like he’s going to
throw that at her, too.
Police Dispatcher: Where is your sister?
Caller: She’s running toward the bathroom.
Police Dispatcher: Is she injured?
Caller: I see some blood on her arm.
Police Dispatcher: Does he have a gun?
Caller: I don’t see a gun.
A nearby police officer arrived on the scene five minutes after the caller telephoned 911. The
police officer found the boyfriend pacing in the front yard and ordered him to sit in the rear seat
of the patrol car. The boyfriend sat in the patrol car, and the officer locked the door from the
outside so that the boyfriend would stay in the car while the officer spoke to the sister.
When the sister saw that her boyfriend was locked in the patrol car, she came out on the porch to
speak with the officer. The sister was in a highly agitated and emotional state, and she had
several fresh cuts on her right arm. The officer asked her how she got the cuts. The sister replied,
“My boyfriend threw a bottle at me which cut my arm.” The sister declined the officer’s offer of
medical assistance but said that she wanted to press charges against her boyfriend. The sister was
in tears throughout her conversation with the officer.
The boyfriend was charged in state court with battery and disorderly conduct. The prosecutor
made every effort to secure the appearance of both the sister and the caller at trial, but when the
trial began, the sister and the caller did not appear.
The prosecutor is attempting to convict the boyfriend without trial testimony from the sister or
the caller. The prosecutor plans to introduce the caller’s statements to the police dispatcher and
to call the officer to testify and to repeat the statements the sister made to him at her house to
prove that the boyfriend attacked the sister.
The 911 recording containing the caller’s statements to the police dispatcher has been properly
authenticated. Defense counsel has objected to the admission of (1) the caller’s statements to the
police dispatcher on the 911 recording and (2) the officer’s testimony repeating the sister’s
statements to the officer (at her house). Defense counsel asserts the following:
a) The caller’s statements to the police dispatcher are inadmissible hearsay.
b) Admission of the caller’s statements to the police dispatcher would violate the
boyfriend’s constitutional rights.
c) The officer’s testimony repeating the sister’s statements is inadmissible hearsay.
d) Admission of the officer’s testimony repeating the sister’s statements would
violate the boyfriend’s constitutional rights.
6
Evidence Question
This jurisdiction has adopted rules of evidence identical to the Federal Rules of Evidence and
interprets the provisions of the Bill of Rights in accordance with relevant United States Supreme
Court precedent.
How should the trial court rule on each defense objection? Explain.
7
NEGOTIABLE INSTRUMENTS QUESTION _____
A man asked a friend for a loan. The friend was willing to make the loan so long as the man paid
interest at a rate that would enable the friend to make a profit on the transaction. After some
discussion, they agreed that the friend would lend the man $4,000, to be repaid one month later
together with interest at a rate two percentage points higher than the “prime interest rate”
charged by First Bank. (First Bank’s prime interest rate is reported daily in the financial press.)
At dinner that evening, the friend handed the man a check for $4,000, payable to his order, that
was drawn on the friend’s account at First Bank. In exchange, the man handed the friend a
document signed by him and dated that day. The document read, in its entirety, as follows: “The
undersigned hereby agrees to pay to bearer the sum of $4,000, plus interest at a rate two
percentage points higher than the prime interest rate charged by First Bank on the date hereof, no
later than one month from the date hereof.”
After dinner, as the two waited for a bus together, they were robbed. The robber took the check
from the man and the document described above from the friend.
The next day, the robber forged the man’s signature on the back of the check and then sold the
check to a check-cashing business, handing the check to the manager of the business in exchange
for $3,500 in cash. The business and its employees acted in good faith and had no reason to
believe that the check did not belong to the robber or that the man’s signature had been forged.
The following day, the robber sold the document that he had stolen from the friend to a local
investor, handing the investor the document in exchange for $2,500 in cash. The investor acted in
good faith and had no reason to believe that the document did not belong to the robber.
A few days later, the manager of the check-cashing business took the check to First Bank,
handed the check to the teller, and asked that the amount of the check be paid. But the teller
refused to pay because the friend had contacted First Bank and stopped payment on the check.
Accordingly, the teller handed the check back to the manager.
On the date on which the document signed by the man called for him to pay, the investor
contacted the man and demanded payment. The man responded that he would not pay because
his promise had been made to his friend, not to the investor, and, moreover, he should not have
to pay because the friend’s check had been stolen from him with the result that he never received
the money that his friend was supposed to loan him.
1. Does the check-cashing business have a right to recover the amount of the check from the
friend? Explain.
2. Is the document signed by the man a negotiable instrument? Explain.
3. Assuming that the document is a negotiable instrument, does the investor have a right to
recover from the man the amount that the man promised to pay in the document he gave
to the friend? Explain.
8
CORPORATIONS QUESTION _____
On February 2, Alice, Bob, and Carla formed ABC Hospitality, LLC (ABC), a member-managed
limited liability company, for the purpose of building, owning, and running a 100-room luxury
hotel in their hometown. ABC soon began to experience unexpected financial problems,
prompting Bob to look for other investment opportunities.
On March 10, Bob told Alice and Carla that, although he would remain as a member of ABC, he
would no longer contribute any capital to ABC, and he was also becoming a co-owner of the
Metro Inn, an existing 200-room hotel in the same town near the ABC hotel project. Alice and
Carla objected to Bob’s plan, fearing that he might put the interests of the Metro Inn ahead of his
existing obligations to ABC. In response, Bob cited § 5.1 of ABC’s Operating Agreement, which
states as follows:
Members of ABC shall not in any way be prohibited from or restricted in managing,
owning, or otherwise having an interest in any other business venture that may be
competitive with the business of ABC.
Shortly after Bob became a co-owner of the Metro Inn, ABC’s financial situation worsened.
Alice and Carla worried that ABC would not be able to pay a bill it owed to its concrete supplier.
Alice proposed to pay the concrete supplier’s bill from her own personal funds and then obtain
reimbursement from ABC once the hotel project was completed. Alice wanted to do this so that
she could file a personal financial statement which underreported her assets and so enable her
son to qualify for student financial aid. Carla agreed to this proposal. Alice and Carla also agreed
to alter ABC’s financial records so that it would appear as if ABC had paid the concrete
supplier’s bill out of its own accounts, without showing the obligation to reimburse Alice for that
amount.
In the weeks following Alice’s payment to the concrete supplier, several other of ABC’s bills
became due. Alice tried to pay as many of these bills as she could using her personal funds, but
despite her best efforts, it soon became clear that ABC was rapidly approaching insolvency. On
August 15, the hotel’s designer left a message for Carla seeking payment of an overdue bill.
Alice and Carla were concerned about the solvency of the company. Without responding to the
designer, Alice and Carla, acting with Bob’s consent, sold all of ABC’s property and remaining
assets. Alice and Carla each kept one-third of the sale proceeds and gave the remaining one-third
to Bob. They did not file articles of dissolution with the state. When the designer later called
Carla again about the bill, she responded that ABC had been “dissolved” and that no payment
would be forthcoming.
1. Did Alice and Carla have any legal basis to object to Bob’s co-ownership of the Metro
Inn? Explain.
2. Under what theory or theories could Alice, Bob, or Carla be personally liable to the
designer? Explain.
9
CONTRACTS QUESTION _____
On May 1, a manufacturer and a chef met at a restaurant trade show. The manufacturer showed
the chef some carving knives that were on sale for $100 each. After examining the knives, the
chef said, “I love these knives! I’ll take 10 of them. Please send them to my restaurant within the
month. As soon as I receive them, I’ll send you a check for $1,000.” The manufacturer said, “I’ll
ship the 10 knives to your restaurant in a few weeks,” and he took the chef’s address for shipping
purposes.
On May 15, the manufacturer sent six knives to the chef. Enclosed in the shipping box was a
document on the manufacturer’s letterhead that stated in its entirety: “It is a pleasure to do
business with you. Enclosed, pursuant to our agreement, are six knives. Please remit $600 at
your earliest convenience.”
On May 17, the chef sent the manufacturer a check for $600 and included in the envelope an
unsigned note to the manufacturer, handwritten on plain paper, requesting the remaining four
knives. The manufacturer did not respond to the note.
The knives were particularly well-suited for the chef’s uses, and the $100 price was a bargain, so
the chef was very eager for the manufacturer to deliver the remaining four knives. On June 17,
the chef wrote to the manufacturer claiming that the manufacturer was contractually bound to
sell the chef 10 knives and that the manufacturer had breached that contract by furnishing only 6
knives. The manufacturer did not reply to the chef’s letter.
Is there an enforceable contract against the manufacturer that binds him to sell 10 knives to the
chef? Explain.
10
REAL PROPERTY QUESTION
Two years ago, a builder constructed a house for a woman and conveyed that house to her for
$300,000 at the closing by a warranty deed, which was promptly recorded. The sale contract
contained no express warranties relating to the condition of the house. To finance the purchase,
the woman borrowed $200,000 from a local bank secured by a mortgage on her new house. The
mortgage note provided that in the event of the woman’s failure to make two consecutive
monthly mortgage payments, the balance would become immediately due and payable. The
mortgage was promptly recorded.
One year ago, the woman accepted a new job and moved. At that time, her house was worth
$360,000 and there was a balance on the mortgage of $195,000. She sold the house to a man and
delivered a quitclaim deed to him in exchange for $160,000. The quitclaim deed was promptly
recorded and made no reference to the woman’s mortgage obligation. The mortgage obligation
was not discharged at the closing. However, the man immediately began to make the woman’s
monthly mortgage payments to the bank after the closing.
Nine months ago, water seeped into the basement of the house during a major storm, causing
substantial damage. It is undisputed that the seepage was due to defective concrete used by the
builder and not to any negligence on the builder’s part. The man called the builder, told him
about the seepage, and demanded that the builder fix the concrete. The builder responded:
“That’s your problem.” The man then repaired the concrete at a cost of $80,000.
Thereafter, the man sued the builder to recover the $80,000 he had spent to repair the concrete.
While the case was pending, the man stopped making mortgage payments. The bank sued the
man to foreclose on the mortgage and, if necessary, obtain a deficiency judgment against him on
the note if the sale proceeds were insufficient to discharge the mortgage debt. The man has
joined the woman as a third-party defendant in the lawsuit.
1. Is the man likely to prevail against the builder to recover the $80,000 he spent to repair
the concrete? Explain.
2. Is the man personally liable for the outstanding balance on the mortgage note between the
woman and the bank? Explain.
3. If the bank is successful in its foreclosure action, will the man be able to recover damages
from the woman? Explain.
11
DECEDENTS’ ESTATES QUESTION
Twenty years ago, John and Mary were married. One month before their wedding, John and
Mary signed a valid prenuptial agreement in which each of them waived “any property rights in
the estate or property of the other to which he or she might otherwise be legally entitled upon the
termination of their marriage by death or divorce.”
Seventeen years ago, John executed a valid will, which provided as follows:
I, John, leave my entire estate to my wife, Mary. However, if I should hereafter have
children, then I leave three-fourths of my estate to my wife, Mary, and one-fourth of my
estate to my children who survive me, in equal shares.
Fifteen years ago, John had an extramarital affair with Beth, who gave birth to their child, Son.
Both Beth and John consented to Son’s adoption by Aunt. At the time of the adoption, Beth,
John, and Aunt agreed that Son would not be told that he was the biological child of Beth and
John.
Three years ago, Aunt died, and Son moved into John and Mary’s home. At that time, John
admitted to Mary that he had had an extramarital affair with Beth which had resulted in Son’s
birth.
Three months ago, Mary filed for divorce. Nonetheless, she and John continued to live together.
One month ago, before John and Mary’s divorce decree was entered, John was killed in a car
accident. John’s will, executed 17 years ago, has been offered for probate. John’s will did not
designate anyone to act as the personal representative of his estate.
John was survived by Mary, Son, and John’s mother.
1. To whom should John’s estate be distributed? Explain.
2. Who should be appointed as the personal representative of John’s estate? Explain.
12
July 2013 MEE
ANALYSES
Federal Civil Procedure
Agency and Torts
Family L aw
Evidence
Negotiable Instruments
Corporations
Contracts
Real Property
Decedents’ Estates
FEDERAL CIVIL PROCEDURE ANALYSIS
(Federal Civil Procedure I.A.1., B. & E.)
ANALYSIS
Legal Problems
(1)(a) What requirements must be met for a federal court to have jurisdiction in a case
based on diversity?
(1)(b) For purposes of diversity jurisdiction, is the woman a citizen of State B, where she
maintains a home and conducts her farming business, or of State A, where she has lived
for most of her life and to which she returns for several months each year, and which she
calls “home”?
(1)(c) For purposes of diversity jurisdiction, is the distributor a citizen of State A (where
it has its food processing, warehousing, and distribution facilities), State B (where it has
its corporate headquarters), and/or State C (where it is incorporated)?
(2) Is venue proper in State A, even though the acts and omissions giving rise to the
woman’s claim occurred in State B and the distributor’s corporate headquarters and state
of incorporation are outside State A?
DISCUSSION
Summary
Because the woman’s claim is based on state law and the amount in controversy exceeds the
jurisdictional amount ($75,000), the key issue in determining whether subject-matter jurisdiction
exists is whether the plaintiff, the woman, and the defendant, the distributor, are citizens of
different states. The woman is most likely a domiciliary and therefore a citizen of State A, where
she has lived continuously all her life and which she still calls “home.” While an argument can
be made that she has become a citizen of State B, the fact that she spends the majority of her
time each year in State B and treats it as her residence for certain purposes probably does not
amount to an abandonment of her State A domicile under the circumstances.
For diversity purposes, corporations may have dual citizenship. The distributor is a
citizen of State C, where it is incorporated. The distributor is also a citizen of State B, the state
where it has its principal place of business. Because the distributor is a citizen of States B and C,
and the woman is probably a citizen of State A, complete diversity exists and the court should
not dismiss the action for lack of jurisdiction.
Assuming that diversity jurisdiction exists, venue is proper in State A, and the court
should not dismiss for improper venue. In an action founded solely on diversity, venue is proper
in a district in which the defendant resides. Corporate defendants are deemed to “reside” in any
state where they are subject to personal jurisdiction. Although the distributor is incorporated in
State C and its headquarters are in State B, it conducts sufficient business activities to subject it
to personal jurisdiction in State A, and thus is deemed to reside there for venue purposes.
15
Federal Civil Procedure Analysis
Point One(a) (10%)
A federal court has jurisdiction of a complaint based on state law if the amount in controversy
exceeds $75,000 and there is complete diversity between the parties. Here, the amount-in-
controversy requirement is satisfied.
Federal district courts have original jurisdiction over state-lawbased suits involving citizens of
different states where the amount in controversy exceeds $75,000. 28 U.S.C. § 1332. In
determining the amount in controversy, a court will accept the plaintiff’s good faith allegations
as to damages unless it appears to a legal certainty that the plaintiff cannot recover that amount.
St. Paul Mercury Indemnity Co. v. Red Cab Co., 303 U.S. 283 (1938). Here, the woman’s claim
for $400,000 in damages appears to have been made in good faith, given that the woman had
earned $80,000 a year in the first two years of the contract and that the distributor breached the
contract with eight years remaining. Certainly, there is nothing to suggest that the woman will
recover less than $75,000 to a legal certainty. Accordingly, the key issue is whether the woman
and the distributor are citizens of different states.
Point One(b) (35%)
The woman is probably a citizen of State A. But an argument can be made that she has acquired
a new domicile in State B and therefore has become a citizen of State B.
To determine an individual U.S. citizen’s state citizenship for diversity purposes, courts look to
the state of the individual’s domicile when the complaint was filed. Newman-Green, Inc. v.
Alfonzo-Larrain, 490 U.S. 826, 83031 (1989). A person’s domicile is generally determined by
two elements: (1) residence in a state, and (2) an intent to remain in that state. In cases like this
one, where an individual resides for extended periods in more than one state, the determination
of domicile often turns on “a complex inquiry into an individual’s intent.” Ner Tamid
Congregation of North Town v. Krivoruchko, 620 F. Supp. 2d 924, 931 (N.D. Ill. 2009). Courts
look at a variety of facts, none of which is dispositive on its own (and many of which can be
manipulated by the individual concerned) in an attempt to determine the individual’s true, fixed,
permanent home and the place to which the individual intends to return when absent. Relevant
facts include the individual’s residence, the location of the person’s property, the person’s voting
behavior, the location of bank accounts, the individual’s memberships and personal associations,
automobile registration, place of employment, and other matters. 13E CHARLES ALAN WRIGHT,
ARTHUR R. MILLER & EDWARD H. COOPER, FEDERAL PRACTICE AND PROCEDURE § 3612 (3d ed.
2009).
It is unclear whether the woman is domiciled in State A or State B. The woman was
clearly a domiciliary of State A, the only state in which she had ever lived, until she bought the
farm and farmhouse in State B. Despite her purchase of the State B farm, she maintains many
social connections with State A, continues to secure medical and dental treatment in State A,
returns to State A to live every winter, and continues to vote and attend church there. One could
argue that these facts show that the woman never intended to abandon her State A domicile, but
always retained the intent to return there.
On the other hand, the woman did acquire property in State B, she moved many of her
personal belongings there, and she lives there for most of the year. Moreover, she took several
steps that suggest her intent to establish a State B domicile, including registering her car there,
obtaining a State B driver’s license, filing state income taxes only in State B, and listing State B
as her residence on her federal tax returns.
16
Federal Civil Procedure Analysis
Although the matter is debatable, the facts suggest that the woman did not move to State
B with the requisite intent to remain there. Instead, it appears that she always intended that State
A would remain her home, although her livelihood requires her presence in State B for much of
the year. In cases of this sort, some courts seek to determine the “center of gravity” of an
individual’s life. See, e.g., Galva Foundry Co. v. Heiden, 924 F.2d 729 (7th Cir. 1991). Despite
the fact that the woman lives in State B for two months more of the year than she lives in State
A, it appears that the “center of gravity” of her life is State A, and that a court would conclude
that she is a domiciliary and therefore a citizen of State A.
[NOTE: Examinees who demonstrate an understanding of the complexity of this issue,
and who use the key facts to develop a cogent argument, should receive full credit regardless of
their conclusion on this point.]
Point One(c) (25%)
The distributor is a citizen of both State C, where it is incorporated, and State B, where it has its
principal place of business (i.e., where its corporate headquarters are located).
By statute, a corporation is a citizen both where it is incorporated and where it has its principal
place of business. 28 U.S.C. § 1332(c)(1). The distributor is incorporated in and therefore is a
citizen of State C. So the question is where the distributor has its principal place of business.
Until the Supreme Court’s decision in Hertz Corp. v. Friend, 559 U.S. 77 (2010), the
federal courts were divided about how to identify a corporation’s principal place of business. See
generally JACK H. FRIEDENTHAL, MARY KAY KANE & ARTHUR R. MILLER, CIVIL PROCEDURE
36–37 (4th ed. 2005). Some courts used a nerve-center test that focused on where the
corporation’s decision-making authority was located. Other courts applied a corporate-
activities/operating-assets test that focused on where a corporation’s business activities occurred.
In Hertz Corp., the Supreme Court decided that a corporation’s principal place of
business for diversity purposes would be deemed to be “the place where a corporation’s officers
direct, control, and coordinate the corporation’s activities,” its so-called “nerve center.” 559 U.S.
at 9293. In this case, the distributor’s corporate headquarters are in State B, its top officers have
their offices and staff in State B, and its decisions regarding the contract with the woman were
made in State B. All of these facts suggest that State B is the place where the corporate officers
“direct, control, and coordinate” the distributor’s activities, and that State B is therefore the
distributor’s principal place of business.
The distributor, therefore, is a citizen of State B (as well as of State C). If the woman is a
domiciliary of State A (see Point One(b)), there is complete diversity of citizenship and the
motion to dismiss for lack of jurisdiction should be denied.
[NOTE: If an examinee previously concluded that the woman is a domiciliary of State B,
then that examinee should also conclude that the action must be dismissed for lack of subject-
matter jurisdiction because there would not be diversity of citizenship between the woman and
the distributor.]
Point Two (30%)
Because the distributor has sufficient contacts with State A to be subject to personal jurisdiction
there, the distributor resides in State A for purposes of venue.
The woman brought this action in the United States District Court of State A. The issue,
therefore, is whether a State A venue can be justified under the general federal venue statute.
When, as here, a federal court’s jurisdiction is based on diversity, 28 U.S.C. § 1391(b) provides
17
Federal Civil Procedure Analysis
that venue is proper (1) in any district where a defendant resides if all defendants reside in the
same state, or (2) in any district where a substantial part of the acts or omissions giving rise to
the action occurred, or (3) in any district where any defendant is subject to personal jurisdiction,
if there is no other district in which the action might be brought.
A State A venue is appropriate if the distributor “resides” in the District of State A for
purposes of the venue statute. According to 28 U.S.C. § 1391(c), a corporation resides where it is
subject to personal jurisdiction at the time the action is commenced. So the question is whether
the distributor is subject to personal jurisdiction in State A.
Rule 4(k) indicates that a federal district court may exercise personal jurisdiction to the
same extent as a state court of general jurisdiction in the state where the district court sits. State
A’s long-arm statute extends jurisdiction as far as the Due Process Clause of the 14th
Amendment allows. The Due Process Clause permits the exercise of personal jurisdiction over
out-of-state defendants like the distributor if they have minimum contacts with the forum state
such that the exercise of jurisdiction would not offend traditional notions of fair play and
substantial justice. International Shoe Co. v. Washington, 326 U.S. 310 (1945). The nature of the
contacts required depends on whether the suit arises out of or is related to the defendant’s
contacts with the forum state.
Here, the distributor maintains its business facilities in State A, including food
processing, warehousing, and distribution facilities. Regardless of whether the woman’s cause of
action is deemed to be related to those contacts, these continuous and substantial business
connections with State A would warrant a State A court to exercise jurisdiction over the
distributor. Moreover, requiring the distributor to defend this action in State A is not likely to be
viewed by the court as unfair or unreasonable. Whether the exercise of personal jurisdiction is
fair and reasonable depends on various considerations such as the burden on the defendant of
having to litigate in the forum state, the plaintiff’s interest in a convenient forum for obtaining
relief, the state’s interest in providing a forum, and the interests of the interstate judicial system
in efficient resolution of disputes. Burger King Corp. v. Rudzewicz, 471 U.S. 462, 477 (1985).
Given modern transportation and communication, the distributor’s burden of defending in a state
where it does continuous and substantial business is minimal even if the suit is unrelated to the
distributor’s business in that forum.
While the woman lives in State B for most of the year, the State A federal court is located
close to her home in the city, where she lives for part of the year. Furthermore, because the
woman is a part-year resident of State A, State A has some interest in providing a forum where
she can obtain redress for legal harms. On the other hand, adjudication in State A is not
particularly efficient. Aside from the woman, who resides in State A for part of the year, it
appears that the bulk of the evidence and witnesses relevant to her case will be located in State
B. Nonetheless, that fact alone would not be enough to defeat State A’s jurisdiction over the
distributor in this case.
Because State A could assert personal jurisdiction over the distributor at the
commencement of the action (had the issue been raised), the distributor will be deemed to
“reside” in State A, rendering the District of State A an appropriate venue under 28 U.S.C.
§ 1391(b)(1). The motion to dismiss for improper venue should be denied.
[NOTE #1: By failing to raise the defense of personal jurisdiction in its motion, the
distributor waived the defense. See FED. R. CIV. P. 12(b) & (h). However, the venue statute asks
whether the distributor was subject to personal jurisdiction at the commencement of the suit, so
the distributor’s waiver of possible personal jurisdiction defense does not affect the venue
inquiry. An examinee should receive credit for a personal jurisdiction analysis ONLY if it is
offered in the context of determining where the distributor “resides” for venue purposes.]
18
Federal Civil Procedure Analysis
[NOTE #2: Here, neither the second nor the third basis for venue is applicable. As to the
second basis for venue (a district where a “substantial part” of the relevant acts occurred), the
woman’s claim is based on an alleged breach of contract. The facts state that this contract was
negotiated in State B, that performance occurred in State B, and that the decision to stop
performing on the contract was made in State B. It does not appear that any of the acts or
omissions giving rise to the woman’s claim occurred in the District of State A.
The third basis for venue is applicable only if there is no other district where the case
could be brought. Here, as just noted, the actions giving rise to the woman’s claim took place in
State B, and there were no impediments to her bringing the claim there. Because there is another
district where suit could have been brought, a State A venue cannot be based on the absence of
such a district.]
19
AGENCY AND TORTS ANALYSIS ____
(Agency II.B.; III. / Torts II.E.1. & F.1.)
ANALYSIS
Legal Problems
(1) Was the employee acting within the course and scope of his employment when he
authorized the foreman to lift the cable?
(2) Did the telephone company cloak the employee with apparent authority that was
relied upon by the foreman, resulting in his injury?
(3) Were the employee’s actions the proximate cause of the foreman’s injuries?
DISCUSSION
Summary
The telephone company is liable to the foreman for the telephone company employee’s
negligence if the employee was acting within the course and scope of his employment with the
telephone company when he authorized the foreman to direct the crew to stretch the cable over
the highway. Here the employee probably was. Even if the employee was acting outside the
scope of his employment with the telephone company, the telephone company would be liable to
the foreman if the foreman relied on statements or conduct by the employee that were within the
employee’s apparent authority. Although the type of harm the foreman suffered (burns) was
different from the harm risked by the employee’s actions (being stuck by the cable), the
employee’s actions are probably the proximate cause of the foreman’s injuries.
Point One (35%)
The telephone company is liable for torts committed by its employee while acting within the
course and scope of his employment with the telephone company. Because the employee was
motivated by a desire to serve the telephone company, he was acting within the scope of his
employment.
Under the doctrine of respondeat superior, employers are liable for the actions of an employee
when the employee is acting within the scope of his employment. See RESTATEMENT (THIRD) OF
AGENCY § 2.04.
Here, the employee had no express authority to authorize the raising of the cable. His
only duty was to determine whether a cable was down and, if so, to assess the damage and report
to the telephone company. However, the employee’s unauthorized conduct may fall within the
scope of his employment. See RESTATEMENT (THIRD) OF AGENCY § 7.07 cmt. c (“conduct is not
outside the scope of employment merely because an employee disregards the employer’s
instructions”). Section 7.07 of the Restatement (Third) of Agency states that “[a]n employee acts
within the scope of employment when performing work assigned by the employer or engaging in
a course of conduct subject to the employer’s control. An employee’s act is not within the scope
of employment when it occurs within an independent course of conduct not intended by the
employee to serve any purpose of the employer.”
20
Agency and Torts Analysis
Here, the employee’s action (authorizing the raising of the cable) was motivated by a
desire to protect the cable from further damage. Thus, the employee was clearly acting with the
intent to serve the telephone company’s purposes. The cable which ultimately caused the injury
to the foreman was the property of the telephone company. The telephone company should have
reasonably expected that when the employee arrived at the scene he would take necessary action
to prevent further damage to its property if he were in a position to do so. Although it is not
certain, the telephone company is probably liable to the foreman under the doctrine of
respondeat superior.
Point Two (30%)
The telephone company could also be liable to the foreman under an apparent authority theory.
A principal will sometimes be liable for torts committed by its agents even if the conditions of
respondeat superior liability (i.e., an employer/employee relationship and conduct within the
scope of employment) are not satisfied.
For example, a principal can be liable if the principal was negligent or reckless in the
selection of the agent. See RESTATEMENT (THIRD) OF AGENCY § 7.05(1). Nothing in the facts
suggests that the telephone company was negligent in its hiring of the employee.
A principal can also be liable for an agent’s torts if the principal has a special relationship
with the injured person that imposed a special duty on the principal to take care to protect against
the risk that the agent would harm the injured person. See id. § 7.05(2). Nothing on these facts
suggests that the telephone company had any such special relationship with the foreman.
Finally, a principal is liable for a tort committed by an agent when the agent “appears to
deal or communicate on behalf of [the] principal and the agent’s appearance of authority enables
the agent to commit [the] tort . . . .” See RESTATEMENT (THIRD) OF AGENCY § 7.08, cmt a. On the
facts of this problem, the employee had apparent authority vis-à-vis the foreman. “Apparent
authority . . . is created by a person’s manifestation that another has authority to act with legal
consequences for the person who makes the manifestation, where a third party reasonably
believes the actor to be authorized and the belief is traceable to the manifestation.” Id. § 3.03. An
agent has apparent authority with respect to a third party only “when the third party reasonably
believes that the agent . . . has authority to act on behalf of the principal.” Id. § 7.08, cmt b. Here,
the telephone company made manifestations to the foreman about the employee’s authority when
it sent its employee to the job site in a company vehicle. This would give the foreman a basis to
believe that the employee was acting on the telephone company’s behalf.
The principal is liable under an apparent authority theory only when the injured person’s
belief in the agent’s authority “enables the agent to commit the tort.” As noted, the employee
arrived at the job site in the telephone company’s vehicle. This was sufficient to create a
reasonable belief that the employee was acting on behalf of the company on all matters relating
to the fallen cable. As a result of this reasonable belief, the foreman, acting on the employee’s
authorization, raised the cable, creating the circumstances leading to his injury.
Thus, on an apparent authority theory the company could be liable assuming respondeat
superior did not apply. See Point One.
Point Three (35%)
The employee’s acts were probably the proximate cause of the foreman’s injuries.
In a negligence action, a defendant is liable only if his conduct was the proximate cause of the
plaintiff’s injury. Here, it is likely that a court would find that the foreman’s harm was
proximately caused by the employee’s negligence.
21
Agency and Torts Analysis
Even when a defendant is negligent, his conduct must have
“such an effect in producing the harm as to lead reasonable men to regard it as a cause,
using that word in the popular sense, in which there always lurks the idea of
responsibility, rather than in the so-called ‘philosophic sense,’ which includes every one
of the great number of events without which any happening would not have occurred.”
RESTATEMENT (SECOND) OF TORTS § 431, cmt. a.
Intervening actors or events that produce harm different in kind from that which one
would normally anticipate may break the chain of causation and lead a court to conclude that the
defendant’s acts are not the proximate cause of the plaintiff’s injury. Thus, a plaintiff who
negligently exceeds a speed limit and therefore happens to be on the spot where a tree falls
during a violent windstorm may still recover for injuries caused by the tree (see Berry v. Sugar
Notch Borough, 43 A. 240 (Pa. 1899)), and a defendant who negligently drops a passenger off at
the wrong train station is not liable for the passenger’s burns caused by a malfunctioning
kerosene lamp in the hotel where she is forced to stay for the night. See Central Georgia Ry. Co.
v. Price, 32 S.E. 77 (Ga. 1898). In evaluating whether intervening acts break the chain of
causation, courts typically analyze both their foreseeability and their degree of dependence on
the defendant’s negligence. See RESTATEMENT (SECOND) OF TORTS § 431.
Here, it was entirely foreseeable that a vehicle would strike the raised cable, causing
impact damage to cars and persons within the range of the cable’s fall; that is what made the
employee’s conduct negligent. Given that the cable was stretched over a highway, it was also
foreseeable that a car struck by a cable would go out of control and cause injuries to other
vehicles or bystanders, and that is exactly what happened here: the driver lost control of the car,
hitting another vehicle and thereby causing serious personal injury to a bystander, the foreman.
Although the source of the foreman’s injuries (burns from hot asphalt) would not normally be
anticipated from the fall of a cable, the foreman was within the zone of foreseeable risk and
suffered injuries in the course of a foreseeable, reasonably direct chain of circumstances. A court
is thus likely to find that the employee’s negligence was the proximate cause of the foreman’s
injuries.
22
FAMILY LAW ANALYSIS
(Family Law II.B; IV.A.; VI.E.)
ANALYSIS
Legal Problems
(1)(a) On what, if any, basis could a court enjoin the mother from making contributions
from her future paychecks to the religious group?
(1)(b) On what, if any, basis could a court order one parent to follow the child-rearing
preferences of the other parent when both parents and their child are living together in an
intact family?
(1)(c) On what, if any, basis could a court order a parent to cooperate in ensuring that a
child obtains medical treatments necessary to protect the child’s health and life?
(2) In a divorce action, could a court grant custody to one parent based on the other
parent’s religious practices when those practices pose risks to the child?
DISCUSSION
Summary
There is no basis for a court to order the mother to stop making contributions from her paychecks
to the religious group. Because the mother and father live in a common-law jurisdiction in an
intact household, the father has no management powers with respect to the mother’s earnings.
Although, in most states, the mother has a support obligation toward the father that would enable
a creditor to recover from the mother the value of “necessaries” furnished to the father, the father
may not personally obtain a court order requiring the mother to spend her earnings in a particular
way, nor may a state child welfare agency.
Based on the family privacy doctrine, a court may not order one parent to follow the
child-rearing preferences of the other parent when the parents live together with their child in an
intact family. Thus, a court will not order the mother to take the daughter to skating lessons.
However, based on its neglect jurisdiction, a court could order a parent to take steps to
preserve a child’s health or safety, even when the parent’s actions are religiously motivated. The
proper party to initiate a child neglect action is the state, not a parent. Thus, a court could order
the mother to follow the physician’s medical recommendations and order appropriate services for
the daughter if the mother refused to do so and the state’s child welfare agency filed a neglect
petition on the daughter’s behalf.
Although a court may not deny a parent custody based on a parent’s religious faith, it
may deny custody based on a threat to the child’s health or safety. Because failing to take asthma
medications appears to be life-threatening, if the father were to file a divorce action against the
mother, a court could award the father custody of his daughter if the mother persists in her
refusal to provide the daughter with her asthma medication.
23
Family Law Analysis
Point One(a) (30%)
There is no basis for a court to order the mother to stop making contributions from her paycheck
to the religious group. Because the mother and father live in an intact household in a non-
community-property state, the father has no management powers with respect to the mother’s
earnings. Although the mother has a support obligation toward the father that would enable a
creditor to recover from the mother the value of “necessaries” furnished to the father, the father
could not personally obtain a court order requiring the mother to spend her earnings in a
particular way, nor could a state child welfare agency.
Under the common law, when a woman married, her identity was swallowed up in her
husband’s. As a result of this marital-unity doctrine, a married woman could not own property.
Beginning in the mid-19th century, legislatures began to enact so-called “Married Women’s
Property Acts” that restored to the married woman the rights she had when unmarried, including
the right to acquire, own, or transfer property. By the end of the 19th century, every state had
passed such a statute.
Under the Married Women’s Property Acts, title determines asset ownership and
management rights; “[t]he wife is given the power to contract and full rights to her own earnings
. . . .” HARRY D. KRAUSE AND DAVID D. MEYER, FAMILY LAW IN A NUTSHELL 99 (5th ed. 2007).
Because each spouse has full management rights with respect to his or her earnings, a court may
not overrule the spending decisions of a spouse based on his or her partner’s conclusion that
those decisions are ill-advised.
Marriage does, however, create support obligations and, since the Supreme Court’s
decision in Orr v. Orr, 440 U.S. 268 (1979), those obligations have been gender neutral. Based
on this mutual support obligation, a creditor who has furnished “necessaries” to a husband or
wife may, in most states, sue the spouse of the purchaser and recover on the debt. See KRAUSE
AND
MEYER, supra, at 94–95.
However, the necessaries doctrine is available only to a creditor who has already
provided goods or services. Such a creditor has no power to obtain an order altering a spouse’s
future spending. The doctrine does not go so far as to allow one spouse to enjoin expenditures by
the other because, as here, he or she is concerned that the bills of the family won’t get paid. Nor,
because of the family privacy doctrine (see Point 2), may a spouse who disagrees with his
partner’s spending decisions obtain an order requiring a different spending pattern or obtain a
support award.
A state child welfare agency may intervene in family decision making only when the
decision at issue endangers the well-being of a child or another family member incapable of
protecting his own interests. Here, there is no evidence that the mother’s decision to donate a
portion of her earnings to the religious group endangers her daughter. Thus, the state agency
cannot obtain an order requiring the mother to alter her spending patterns any more than can the
father.
Point One(b) (20%)
Because the mother and father are living with their daughter in an intact family and failure to
take skating lessons does not endanger the daughter, a court may not require the mother to follow
the father’s preference with respect to continuing the daughter’s skating lessons.
American courts have consistently treated the disputes of intact families as private matters that
should be resolved at home. Thus, they have refused to decide such disputes when a spouse has
brought one to court:
24
Family Law Analysis
The inherent jurisdiction of courts of equity over infants is a matter of necessity, . . .
assumed by the courts only when it is forfeited by a natural custodian incident to a broken
home. . . . The judicial mind and conscience is repelled by the thought of disruption of
the sacred marital relationship, and usually voices the hope that the breach may somehow
be healed by mutual understanding between the parents themselves.
Kilgrow v. Kilgrow, 107 So. 2d 885, 88889 (Ala. 1958); see also McGuire v. McGuire, 59
N.W.2d 336 (Neb. 1958). Even when the spouse who comes to court wishes to enforce a
premarital agreement, courts have refused to intervene in the disputes of couples who are living
together. See Kilgrow, supra (refusing to enforce provision of premarital agreement requiring
child’s education at religious school).
Here, the mother and father are still living together with their daughter. Until and unless
they separate, a court will not intervene in their dispute regarding the daughter’s skating lessons
and issue an order supporting the views of either parent.
Again, the state child welfare agency may intervene in family decision making only when
the decision at issue endangers the well-being of a child or another family member incapable of
protecting his own interests. Here, because there is no evidence that the mother’s failure to
continue skating lessons would endanger the daughter, the state agency cannot obtain an order
requiring the mother to continue the lessons any more than can the father.
Point One(c) (30%)
Because failing to take asthma medications poses a serious risk to the daughter’s health and
safety, if the mother’s actions prevent the daughter from receiving her prescribed medications, a
court may find that the daughter is neglected and order the mother to give the daughter her
medications.
Although courts will not intervene in disputes between parents in an intact household, the state,
pursuant to its jurisdiction over child abuse and neglect, may obtain an order overruling a
parental decision and ordering appropriate services, including medical care, whenever the
parental child-rearing decisions endanger the child.
The fact that parental rights are constitutionally protected does not alter this result, even
when the parental choice is religiously motivated: “the power of the parent, even when linked to
a free exercise claim, may be subject to limitation . . . if it appears that parental decisions will
jeopardize the health or safety of the child, or have a potential for significant social burdens.”
Wisconsin v. Yoder, 406 U.S. 205, 23435 (1972). See also Petersen v. Rogers, 433 S.E.2d 770,
775 (N.C. Ct. App. 1993) (although court may not inquire into parent’s general religious beliefs,
it may inquire into “religious practices . . . if such practices may adversely affect the physical or
mental health or safety of the child.”); Jay M. Zitter, Annot., Power of Court or Other Public
Agency to Order Medical Treatment Over Parental Religious Objections for Child Whose Life Is
Not Immediately Endangered, 21 A.L.R. 5th 248 (1994).
[NOTE: Here, a child-neglect action would most likely be triggered by the daughter’s
physician, who is almost certainly a mandated neglect reporter under state law. However, the
father could also file a neglect report. A neglect action is typically commenced after someone
with knowledge of a situation that endangers the child files a report with the appropriate state
agency. State laws typically require health care workers, among others, to report suspected abuse
and neglect. After receiving a report, the agency investigates and files a neglect petition on the
child’s behalf if its investigation confirms the allegations of neglect. If the court finds that a child
25
Family Law Analysis
is neglected, it may order parents to undertake appropriate actions, order a state agency to
provide the child with services in the home, or even remove the child from parental custody and
place her in foster care.]
Point Two (20%)
Although a court may not deny a parent custody based on the parent’s religious faith, it may
deny custody based on a threat to the child’s health or safety. Because failing to take asthma
medications is life-threatening, a court could, in a divorce action filed by the father, award
custody to the father if the court finds that the mother will persist in her refusal to provide the
daughter with her medications.
A child custody contest between parents is decided on the basis of the child’s best interests.
Under the best interests test, the court is free to consider a wide range of factors.
With respect to parental religion, the Establishment Clause forbids a court to favor one
religion over another. However, although “[t]he court cannot evaluate the religion, [it can] . . .
instead evaluate[] the parent’s ability to provide for the physical and mental health needs of the
child.” WALTER WADLINGTON & RAYMOND C. O’BRIEN, FAMILY LAW IN PERSPECTIVE 162
(2001). Because of the sensitivity of the constitutional issues at stake, courts have typically
refused to consider religion unless the evidence shows that the parent’s religious practice would
imperil the child’s well-being. See id. Some courts have additionally required that a custody
order “make the least possible infringement upon the parent’s liberty interests consistent with the
child’s well-being.” Osier v. Osier, 410 A.2d 1027, 1030 (Me. 1980). See also Carl E. Schneider,
Religion and Child Custody, 25 U. MICH. J. L. REFORM 879 (Spring/Summer 1992).
Here, the daughter’s physician has said that failure to take asthma medications is life-
threatening. Given that the daughter is only seven and needs parental supervision in order to
ensure medication compliance, should the father file a divorce action, a court may thus deny the
mother custody based on her refusal to administer the medications, and may award custody to the
father.
26
EVIDENCE ANALYSIS
(Evidence V.A., B., D., F. & L.)
ANALYSIS
Legal Problems
(1) Do the caller’s statements to the police dispatcher fit a hearsay exception?
(2) Would admission of the caller’s statements to the police dispatcher violate the
Confrontation Clause?
(3) Do the sister’s statements to the officer fit a hearsay exception?
(4) Would admission of the sister’s statements to the officer violate the Confrontation
Clause?
DISCUSSION
Summary
The court should allow admission of the caller’s out-of-court statements to the police dispatcher
on the authenticated 911 recording because they fit the hearsay exceptions for “present sense
impressions” and “excited utterances.” The caller’s statements may also fit the exception for
“statements made for medical diagnosis or treatment.”
Admission of these statements would not violate the Confrontation Clause of the Sixth
Amendment to the United States Constitution. Although the caller is unavailable to testify and
the boyfriend will have had no pretrial opportunity to cross-examine her, the caller’s statements
are not testimonial because the evidence demonstrates that the reasonable and objective primary
purpose of both the caller and the police dispatcher was to address an ongoing emergency.
The sister’s statements to the officer fit the hearsay exception for “excited utterances.
However, admission of the sister’s statements through the testimony of the officer would
violate the Confrontation Clause because the sister is unavailable to testify, the boyfriend will
have had no opportunity to cross-examine her, and her statements are testimonial. The sister’s
statements are testimonial because the reasonable and objective primary purpose of the sister and
the officer was not to address an ongoing emergency, but to establish past events potentially
relevant to a later criminal prosecution of the boyfriend.
Point One (20%)
If offered to prove the truth of the matter asserted, that the boyfriend attacked the sister, the
caller’s statements to the police dispatcher are hearsay. However, her statements fit the hearsay
exceptions for “present sense impressions” and “excited utterances,” and probably fit the
exception for statements made for purposes of medical diagnosis.
As an out-of-court statement offered to prove the truth of what the caller told the police
dispatcher (i.e., that her sister’s boyfriend was “out of control,” had thrown a broken beer bottle
at her sister, and that her sister was bleeding), the caller’s statements on the authenticated 911
27
Evidence Analysis
recording are hearsay. However, the court should overrule defense counsel’s hearsay objection
because the caller’s out-of-court statements fit two hearsay exceptions.
First, the caller’s statements fit the hearsay exceptions for “present sense impressions.”
Pursuant to Rule 803(1), a present sense impression is “[a] statement describing or explaining an
event or condition made while or immediately after the declarant perceived it.” FED. R. EVID.
803(1) (emphasis added). In this case, the caller described the events to the police dispatcher as
they were happening.
Second, the caller’s statements also fit the hearsay exception for “excited utterances,”
pursuant to Rule 803(2). An excited utterance is “[a] statement relating to a startling event or
condition, made while the declarant was under the stress or excitement that it caused.FED. R.
EVID. 803(2). Here, the caller telephoned 911 while she was watching her sister’s boyfriend
violently assault her sister. The caller’s statements related to the startling event of the assault on
her sister. Moreover, because the caller made the statements as she watched the assault, she
would have been under the stress caused by the assault on her sister at the time her statements
were made.
Third, some of the caller’s statements probably also fit the hearsay exception for
“statements made for medical diagnosis or treatment.” Statements that fit this exception must
have been “made forand [] reasonably pertinent tomedical diagnosis or treatment; and
describe[] medical history; past or present symptoms or sensations; their inception; or their
general cause.” FED. R. EVID. 803(4). Here, the police dispatcher asked specific questions about
the sister’s injuries and the caller stated that her sister had blood on her arm. Thus, this portion of
the caller’s statement was relevant to the sister’s medical condition and the fact that she might
need medical treatment.
[NOTE: The questions from the police dispatcher are not hearsay because they are not
assertions.]
Point Two (25%)
The Confrontation Clause does not bar the admission of an out-of-court statement when the
objective primary purpose of the interrogation was to address an ongoing emergency.
Even when, as here, an out-of-court statement falls within a hearsay exception, the Confrontation
Clause of the Sixth Amendment of the United States Constitution will sometimes preclude the
admission of the statement at trial. The Confrontation Clause gives a defendant in a criminal case
the right to be confronted by the witnesses against him. U.S. CONST. AMEND. VI (“In all criminal
prosecutions, the accused shall enjoy the right . . . to be confronted with the witnesses against
him.”). In Crawford v. Washington, 541 U.S. 36 (2004), the Supreme Court held that the
admission of out-of-court “testimonial” statements violates a defendant’s right to confrontation if
the witness is unavailable to testify at trial and the defense has not had a prior opportunity to
cross-examine the witness. Id. at 59. In this case, the caller is unavailable because the
prosecutor’s efforts to secure her appearance at trial have been unsuccessful, and the defense has
not had a prior opportunity to cross-examine the caller.
Thus, the critical constitutional question is whether the caller’s statements to the police
dispatcher were “testimonial.” After Crawford, many statements made to police officers in the
course of an interrogation are testimonial. Id. at 5253 (“interrogations by law enforcement
officers fall squarely within that class [of testimonial statements]”). However, in Davis v.
Washington, the Court clarified that when witnesses make statements to the police “under
circumstances objectively indicating that the primary purpose of the interrogation is to enable
police assistance to meet an ongoing emergency,” these statements are not testimonial. See Davis
28
Evidence Analysis
v. Washington, 547 U.S. 813, 822 (2006). More recently, in Michigan v. Bryant, 131 S.Ct. 1143
(2011), the Court elaborated on the “primary purpose of the interrogation” standard, noting that
primary purpose should be determined using “ a combined inquiry that accounts for both the
declarant and the interrogator.Bryant, 131 S.Ct. at 1160. The Bryant Court also specified that
“[a]n assessment of whether an emergency that threatens the police and public is ongoing cannot
narrowly focus on whether the threat solely to the first victim has been neutralized because the
threat to first responders and to the public may continue,” id. at 1158, and that “the duration and
scope of an emergency may depend in part on the type of weapon employed,” id., and/or “[t]he
medical condition of the victim,” id. at 1159.
Here, the caller telephoned the police for assistance while the boyfriend was assaulting
the sister. The sister was injured and bleeding, and the boyfriend could have posed a threat to the
caller and/or first responders. Thus, it is clear that the caller made statements to the police
dispatcher while the emergency was ongoing. This conclusion is reinforced by the fact that the
police dispatcher’s questions were aimed at addressing the ongoing emergency (i.e., the
dispatcher attempted to ascertain the extent of the concurrent violence, the level of danger,
the parties involved, and whether anybody was injured). Based on these facts, the caller and the
police dispatcher both had a primary purpose of resolving the ongoing emergency created by the
boyfriend’s violent physical assault of the sister. Thus, the caller’s statements were not
testimonial and admission of these statements does not violate the Confrontation Clause. See
Bryant, 131 S.Ct. at 1162; Davis, 547 U.S. at 827.
Point Three (25%)
The sister’s statements to the officer fall under the “excited utterance” exception to the hearsay
rule.
The court should overrule defense counsel’s hearsay objection to admission of the sister’s
statements to the officer. Testimony repeating the sister’s out-of-court statements would be
hearsay because the prosecution is seeking to use the statements to prove the truth of the matter
asserted by the sister (that the boyfriend had just attacked her). However, the sister’s statements
would be admissible pursuant to the “excited utterance” exception to the hearsay rule. F
ED. R.
E
VID. 803(2). As discussed above in Point One, an excited utterance is “[a] statement relating to
a startling event or condition made while the declarant was under the stress or excitement that it
caused.” Id. Certainly the boyfriend’s violent attack was a startling event and the sister was
clearly still “under the stress or excitement that it caused” when the officer arrived at her house
five minutes later. The statements were made within minutes of the boyfriend’s violent conduct,
the sister was in a highly emotional and agitated state (she was in tears when she spoke to the
officer), and her statements related directly to the boyfriend’s attack, which was the event that
caused her to be excited and upset.
[NOTE: The sister’s statements here do not fit the hearsay exception for statements made
for the purpose of medical treatment, FED. R. EVID. 803(4), because she specifically declined the
medical assistance offered by the officer.]
Point Four (30%)
The Confrontation Clause prohibits the use of the sister’s statements to the officer because they
are testimonial. The objective primary purpose of this interrogation was not to address an
ongoing emergency, but to establish past events potentially relevant to a later criminal
prosecution.
29
Evidence Analysis
The dispositive question is whether admission of this out-of-court statement would be
precluded under the Confrontation Clause. As discussed above in Point Two, the Supreme Court
defined testimonial out-of-court statements that require confrontation in Crawford, Davis, and
Bryant. Unlike the caller’s statements to the police dispatcher during the boyfriend’s attack, the
sister’s statements to the officer were made when the emergency was over and described what
the boyfriend had done in the past. Although just a short period of time had passed and the sister
was clearly still upset about the boyfriend’s attack, the boyfriend was now locked in the back of
the police car and no longer posed any danger to the sister, the officer, or anyone else.
In 2006, in Davis v. Washington, the Supreme Court focused on the question of whether
the primary purpose of each interrogation was to enable police assistance to meet an ongoing
emergency. In 2011, the Bryant Court significantly elaborated on the operation of the standard in
a range of contexts. The Bryant Court cautioned that “[a]n assessment of whether an emergency
that threatens the police and public is ongoing cannot narrowly focus on whether the threat solely
to the first victim has been neutralized because the threat to first responders and to the public
may continue.” Bryant, 131 S. Ct. at 1158.
Here, the facts indicate that the emergency was resolved by the time the sister made her
statement. See State v. Wright, 726 N.W.2d 464, 476 (Minn. 2007) (finding no ongoing
emergency after alleged assailant was placed in a police car). Under these circumstances, the
primary purpose of the interrogation was not to resolve an ongoing e mergency but to establish
past events potentially relevant to a later criminal prosecution of the boyfriend. See Davis, 547
U.S. at 822. Because the sister’s statements were testimonial, she is unavailable to testify at trial,
and the boyfriend has had no opportunity to cross-examine her, the admission of the sister’s
statements through the trial testimony of the officer would violate the Confrontation Clause.
The facts specify that the prosecutor seeks to admit the caller’s and the sister’s statements
to prove the attack (i.e., as substantive evidence). However, if either statement were admitted for
a non-truth purpose (e.g., to prove the caller’s or the sister’s state of mind), it would not be
barred by the Confrontation Clause.
30
NEGOTIABLE INSTRUMENTS ANALYSIS
(Negotiable Instruments IB.; II.; III.; IV.; V.; VI.)
ANALYSIS
Legal Problems
(1) May a person in possession of a check on which the payee’s indorsement has been
forged recover from the drawer if the drawee refuses to pay the check?
(2) Is a promise to pay to the order of bearer a fixed amount of money plus an interest
rate determined by reference to an external standard a negotiable instrument and, thus, a
note?
(3) Must the maker of a note that is payable to bearer pay a person to whom the note has
been transferred for value if the maker has defenses against the payment obligation of
which the transferee had no notice?
DISCUSSION
Summary
If a check is dishonored, the drawer owes the amount of the check to a person entitled to enforce
it. Because the check was stolen from the man and the man’s indorsement was forged, the check-
cashing business was not a “holder” of the check and was not entitled to enforce it. Thus, the
friend, as drawer of the check, has no obligation to the check-cashing business.
The document that the robber sold to the investor was a negotiable note. Because the note
was payable to bearer, the investor became a holder of the note when the investor gained
possession of it. As a result, the investor is entitled to enforce the note. It can be enforced against
the man because the man was the maker of the note. The investor is also a holder in due course
and is therefore not subject to the man’s defense of never having received the loan.
Point One (30%)
The obligation of the drawer of a check is that, upon dishonor of the check, the drawer will pay
the amount of the check to a person entitled to enforce it. Because the check was payable to the
man, who did not indorse it or transfer it, the check-cashing business will not qualify as a person
entitled to enforce the check, and the friend has no obligation to pay it.
The friend was the drawer of the check. Upon dishonor of a check, the drawer is obligated to pay
the amount of the check to a person entitled to enforce it. Thus, the check-cashing business has a
legal right to recover from the friend only if the business is a person entitled to enforce the
check. UCC § 3-414.
The check-cashing business would be entitled to enforce the check if it was a holder of
the check, or if it was in possession of it and had the rights of a holder. UCC § 3-301.
A holder is a person in possession of a check that is payable to that person or to bearer.
UCC § 1-201(b)(21)(A). Although the check-cashing business had possession of the check, the
check was not payable to that business nor to bearer. As written, the check was payable to
the order of the man. The man didn’t sign the back of the checkhis signature was forged by the
robber. If the man had signed the back of the check, the signature would have qualified as an
31
Negotiable Instruments Analysis
indorsement in blank, which would have had the effect of converting the check to a bearer
instrument. UCC § 3-205(b). Or the man could have indorsed the check to the check-cashing
business, making it payable to that business. But the robber’s forgery of the man’s signature does
not count as the man’s signature. UCC § 3-403(a). As a result, the check is still payable to the
order of the man and the check-cashing business cannot qualify as a holder of it.
The check-cashing business would also be entitled to enforce the check if it had “the
rights of a holder.” UCC § 3-301(ii). A person can obtain the “rights of a holder” by taking the
check by “transfer” from a holder. UCC § 3-203(b). Here, the robber did transfer the check to
the check-cashing business, see UCC § 3-203(a), but the robber was not a holder of the check,
nor did the robber have rights of a holder that he could transfer to the check-cashing business.
The robber was not a holder because the check was not payable to the robber. The robber did not
have the “rights of a holder,” i.e., the man’s rights, because the man did not voluntarily deliver
the check to the robber. See UCC § 3-203(a) (“transfer” of an instrument is a voluntary delivery
for purpose of giving the transferee the right to enforce an instrument); the robber stole the check
and did not become a holder or acquire the rights of a holder. Because the robber did not have
the rights of a holder, the robber’s delivery of the check to the check-cashing business did not
transfer such rights to the business.
Accordingly, the check-cashing business is not a person entitled to enforce the check and
the friend has no obligation to it.
[NOTE #1: Some examinees might mention that the check-cashing business could
theoretically qualify as a person entitled to enforce a check if the business were a person who
was entitled to enforce the check at the time when it was lost, stolen, or destroyed. UCC § 3-
301(iii). But no facts support such an argument.]
[NOTE #2: Some examinees might also argue that the friend has no obligation to pay the
check because it was not technically dishonored, given that it was never presented for payment
by a person entitled to enforce it. This would be a correct analysis, see UCC §§ 3-502(b)(2) and
3-501(a), and an examinee who concludes that the friend has no obligation on the check because
it was not dishonored should receive full credit.]
Point Two (30%)
The document signed by the man and given to the friend was a negotiable instrument because it
was an unconditional promise to pay a fixed amount of money to bearer and contained no other
promises or undertakings. The fact that the note bore interest at a rate requiring reference to an
external source does not prevent it from qualifying as a negotiable instrument.
A negotiable instrument is an unconditional promise or order to pay a fixed amount of money,
with or without interest, if it is payable to bearer or to order and does not state any other
undertaking or instruction. UCC § 3-104(a). A “promise” is a written undertaking to pay money
signed by the person undertaking to pay. UCC § 3-103(a)(12). Based on the facts presented, the
man’s undertaking clearly qualifies as a “promise.” Since the promise was unconditional, it was
payable to bearer, the amount of money was fixed in the promise, and the promise did not state
any other undertakings or instructions, it qualifies as a negotiable instrument. A negotiable
instrument that is a promise qualifies as a “note.” UCC § 3-104(e).
The fact that determination of the amount of interest requires reference to the prime
interest rate of First Bank does not mean that the promise is not for a fixed amount of money.
Interest may be stated at a fixed or variable rate, and may require reference to information not
contained in the instrument. UCC § 3-112(b).
32
Negotiable Instruments Analysis
Point Three (40%)
The investor was a holder of the note, so the man had an obligation to pay the investor the
amount of the note when it was due. The investor qualified as a holder in due course, so
the man’s defense that he never received the amount of the loan could not be raised against the
investor.
As maker of the note, the man had an obligation to pay the amount of the note when due to a
person entitled to enforce it. UCC § 3-412. The investor was a holder of the note because the
investor was in possession of it and it was payable to bearer. UCC § 1-201(b)(21). Accordingly,
the investor qualified as a person entitled to enforce it. UCC § 3-301(i).
If the man’s failure to receive the money from the friend would qualify, as a matter of
contract law, as a defense to his obligation to repay a loan, he could raise that as a defense to his
obligation to pay the investor on the note. But such a defense cannot be raised against a person
who qualifies as a holder in due course. This is because a holder in due course is subject to no
defenses except the so-called “real defenses” listed in UCC § 3-305(a)(1), none of which is
applicable here. The man’s defense that he failed to receive consideration for the note cannot be
asserted against a holder in due course. UCC § 3-305(a)(2), comment 2 (“instruments issued
without consideration or for which promised performance has not been given” can be enforced
by a holder in due course).
On the facts of this problem, the investor meets all requirements to qualify as a holder in
due course. As described above, the investor was a holder. The investor paid $2,500 for the note,
thus taking it for value. The facts state that the investor took the note in good faith and without
notice of any claims, defenses, or defects affecting the note. UCC § 3-302(a). Thus, the investor
was a holder in due course.
As a holder in due course, the investor is not subject to the man’s defense on the note and
has a legal right to recover the amount of the note from the man.
33
CORPORATIONS ANALYSIS __________
(Corporations I.D.; III.; V.D.2.; VI.B.; VIII.F.)
ANALYSIS
Legal Problems
(1) Did Alice and Carla have any legal basis to object to Bob’s co-ownership of the
Metro Inn?
(2) Is the designer entitled to hold Alice, Bob, and/or Carla personally liable to the
designer
(a) because the winding up was improper, or
(b) because of the piercing-the-veil theory?
DISCUSSION
Summary
ABC is a limited liability company (LLC) governed by state LLC law and the terms of its
operating agreement. Alice and Carla most likely do not have any legal basis to object to Bob’s
ownership interest in a competing business because of the express terms of ABC’s operating
agreement.
Alice, Bob, and Carla likely are personally liable to the designer for ABC’s debt up to the
amount each member received from the sale of ABC’s assets because the attempted dissolution
and winding up of ABC was improper. Alternatively, they may be personally liable to the
designer on a piercing-the-veil theory because their siphoning of the proceeds from the sale of
ABC’s assets was inequitable, though not necessarily because the company’s books failed to
reflect Alice’s payment of company debts.
Point One (30%)
Alice and Carla most likely do not have any legal basis to object to Bob’s decision to own an
interest in a competing business in light of the express terms of the ABC operating agreement.
ABC is a limited liability company (LLC). An LLC is a form of business association that
combines many of the most attractive features of corporations and partnerships. Like
corporations, an LLC generally provides its investorscalled “memberswith limited liability
for firm debts. Like general partnerships, LLCs provide members with considerable flexibility in
developing rules for decision making and control. Because LLCs are a relatively new form of
business association bearing many similarities to corporations and partnerships, courts often turn
to either corporate or partnership law to analyze issues in the LLC context.
Here, the issue of whether Alice and Carla have any legal basis to object to Bob’s
ownership of an interest in a competing firm will turn on the duties and obligations that Bob
owes to his fellow LLC members. Courts generally hold that members of an LLC, like partners
in a general partnership, are in a “fiduciary relationship.See McConnell v. Hunt Sports
Enterprises, 725 N.E.2d 1193, 121416 (Ohio Ct. App. 1999). This fiduciary relationship is
defined as a relationship in which members owe one another the duty of utmost trust and loyalty.
Id. As a result, in an LLC, as in a general partnership, direct competition by members would
34
Corporations Analysis
ordinarily be precluded as a violation of the duty of loyalty. Id.; see also ULLCA § 409(b)(3)
(members have a duty to “refrain from competing with the company in the conduct of the
company’s activities before the dissolution of the company”).
Although the question does not state that ABC’s 100-room luxury hotel project and the
200-room Metro Inn would compete, their similarity and proximity in the same town suggest that
they would likely compete. Further, the fact that Alice and Carla are bothered by Bob’s interest
in Metro Inn suggests that competition is a strong possibility.
However, here the LLC operating agreement expressly allows members to have an
interest in a business that competes with the firm. Under most LLC statutes, members of an LLC
can agree to restrict or limit the duty of loyalty, provided the opt-out is specified in the operating
agreement. Thus, the operating agreement controls over the provisions of the statute. In this case,
when they became members of ABC, Alice and Carla agreed to abide by the terms of the
operating agreement, which expressly allow any member (i.e., Bob) to manage, own, and
otherwise have an interest in a business that competes with the firm.
States differ slightly on how the opt-out must be structured in order to be effective. Some
states, including Delaware, provide for total freedom of contract. See Elf Atochem North
America, Inc. v. Jaffari, 727 A.2d 286 (Del. 1999) (applying Delaware LLC Act § 18-1101(b)).
In these states, the terms of the operating agreement govern, unless they conflict with a
mandatory statutory provision designed to protect third parties. Id. at 292. Since Bob’s interest in
Metro Inn involves only the relationship between the contracting membersAlice, Bob, and
Carlathis exception is not implicated.
However, most states that permit opt-outs of the duty of loyalty follow the general
approach set forth in the Uniform Limited Liability Company Act (2006). According to ULLCA
§ 110(d)(1)(C), “If not manifestly unreasonable, the operating agreement may . . . restrict or
eliminate the duty . . . to refrain from competing with the company in the conduct of the
company’s business before the dissolution of the company.” In addition, so long as it is not
“manifestly unreasonable,” the operating agreement may also “identify specific types or
categories of activities that do not violate the duty of loyalty.” Id. § 110(d)(2).
In this case, the operating agreement is not a general waiver of the duty of loyalty, but
specifies the activities that do not violate the duty, namely “managing, owning, or otherwise
having an interest” in any competing business. Further, most courts would likely find this
provision not “manifestly unreasonable.” See McConnell, 725 N.E.2d at 1193.
Whether the provision is “manifestly unreasonable” is a question of fact. Nonetheless,
assuming that Alice and Carla each had notice of the provision when they signed the operating
agreement, it will be difficult for them to claim surprise. The provision clearly allowed each
member to manage, own, or otherwise have an interest in a competing business. This type of
provision, which is typical in commercial real estate deals, appears reasonable. Many investors
will likely want to be free to invest and exercise ownership rights in other similar real estate
ventures in the same area or town.
Point Two(a) (35%)
The designer may be able to hold Alice, Bob, and Carla proportionately liable, up to the amount
of the proceeds each received in dissolution, for ABC’s debt to the designer because the winding
up of ABC following dissolution was improper.
Like a corporation, the LLC provides limited liability for its members. ULLCA § 304(a). This
means that members of an LLC may not generally be held personally liable for the debts of the
firm. The general rule, however, does not apply in a few situations, including when (1) the
35
Corporations Analysis
proper procedures for dissolution and winding up have not been followed, and (2) a court decides
to “pierce the LLC veil.”
Here, the designer may be able to hold Alice, Bob, and Carla liable for ABC’s debts on
the ground that the winding up of ABC was improper. In the absence of a contrary agreement or
court order, dissolution of an LLC requires consent of all the members. ULLCA § 701(a)(2). In
this case, the requirement of unanimous consent has been met since Bob gave his consent to
Carla and Alice selling all of ABC’s assets and property. Although ABC was properly dissolved
by the unanimous consent of the three members, see id., the problem was that the winding up
process was flawed.
After dissolution, as part of the winding up process, the LLC must provide notice of the
dissolution to creditors so that they can make claims against the dissolving entity. ULLCA
§§ 702(b)(1), 703 (notice procedure for “known claims”); see also New Horizons Supply
Cooperative v. Haack, No. 98-1865, 1999 WL 33499 (Wis. Ct. App. Jan. 28, 1999); LARRY E.
RIBSTEIN & ROBERT R. KEATINGE ON LIMITED LIABILITY COMPANIES § 11.6 (2d ed. 2004). The
notice sent to creditors must outline the steps that are necessary for enforcing their claims.
ULLCA § 703(b). Here, it is clear from Carla’s statement that the designer’s claim was known at
the time of dissolution and that ABC did not provide sufficient notice to the designer of the
dissolution.
When such procedures have not been followed, and if the LLC’s assets have been
liquidated and distributed to the members, then a creditor’s claim against the LLC may be
enforced against each of the LLC members to the extent of the member’s proportionate share of
the claim or to the extent of the assets of the LLC distributed to the member in liquidation,
whichever is less. ULLCA § 704(d)(2). However, a member’s total liability for creditor claims
may not exceed the total value of assets distributed to the member in dissolution. Id.
While the facts state that no articles of dissolution were filed with the state, the filing of
such notice is not required for a proper dissolution of an LLC. See ULLCA § 702(b)(2)(A). As a
result, the designer can recover proportionately from Alice, Bob, and Carla personally for ABC’s
debt up to the amount that each member received in the improper winding up after dissolution.
Point Two(b) (35%)
The designer may be able to hold Alice, Bob, and Carla jointly and severally liable for the debt it
is owed from ABC on a “piercing the LLC veil” theory.
As an alternative basis for holding Alice, Bob, and Carla personally liable for ABC’s
debts, the designer could pursue a claim for piercing the LLC veil. Piercing the veil in the LLC
context is a common-law equitable doctrine that prevents members from hiding behind the veil
of limited liability in situations where they have improperly used the LLC form. See Kaycee
Land and Livestock v. Flahive, 46 P.3d 323 (Wy. 2002). To pierce the LLC veil, courts generally
apply the same analysis and factors as in cases where a third party attempts to pierce the
corporate veil and hold shareholders of a corporation personally liable for firm debts. This
requires analyzing whether members have treated the LLC as a separate entity or whether it has
instead become the “alter ego” of the members. If the latter is true, members will not be
permitted to “enjoy immunity from individual liability for the LLC’s acts that cause damage to
third parties.” Id. at 327. Each member for whom the veil is pierced becomes subject to joint and
several liability to the creditor bringing the claim.
Among the factors that courts have used under the alter ego doctrine in the corporate
context is whether the “dominant shareholder siphoned corporate funds,” a factor also used in the
LLC context. See Wilson v. Thorn Energy, LLC, 787 F. Supp. 2d 286, 295 (S.D.N.Y. 2011). In
36
Corporations Analysis
arguing for piercing the LLC veil, the designer will likely assert that the three members of ABC
improperly distributed to themselves the proceeds of the asset sale, thus disregarding the
separateness of firm and personal finances. The use of business funds for personal use is a
frequent factor for piercing in both the corporate and LLC contexts.
Alice, Bob, and Carla, all of whom received the proceeds of the asset sale when ABC
was liquidated, will have a difficult time arguing that this distribution was not inequitable.
Although they did not misrepresent ABC’s weak financial situation to the designer, their
siphoning of assets would seem to be conclusive. See Comment to ULLCA § 304(b) (“key
piercing factor” is the disregard of LLC’s “economic separateness” from its owners, such as
when owner “writes checks on the company’s account for personal expenses”). In fact, in the
corporate context when courts determine that business assets have been intermingled and used
for personal use, piercing follows in about 85 percent of the cases. See Robert B. Thompson,
Piercing the Corporate Veil: An Empirical Study, 76 CORNELL. L. REV. 1036, 1063 (1991).
In addition, courts have identified the intermingling of personal and business funds as a
factor for piercing in the corporate context, as well as in the LLC context. See Sea-Land
Services, Inc. v. Pepper Source, 941 F.2d 519 (7th Cir. 1991). The designer might point to
Alice’s use of personal funds to pay the concrete supplier and other creditors of ABC, expecting
that ABC would later reimburse her. The designer might further assert that Carla and Alice
engaged in “fraud” by altering ABC’s financial statements to cover up Alice’s personal
payments, as further justification for piercing the LLC veil on equitable grounds.
Nonetheless, Alice, Bob, and Carla may be able to argue that the failure to follow
corporate formalities in the accounting of Alice’s “advances of funds” to the LLC (that is, her
undisclosed payment of firm debts, subject to later reimbursement) is not enough to pierce the
veil. The LLC statutes in most states, including those that follow the Uniform Limited Liability
Company Act, provide that “the failure of a limited liability company to observe any particular
formalities relating to . . . management of its activities” is not a proper ground for imposing
personal liability on members for debts of the firm. See ULLCA § 304(b). Moreover, because
Alice and Carla were trying to keep the business afloat during difficult financial times, Alice’s
undisclosed advances of funds did not harm the firm’s creditors, but actually worked to their
advantage. In this sense, Alice’s payments could be equated to additional capital contributions to
the firm, not the improper siphoning of assets. Thus, taken alone, the off-the-books advances by
Alice to ABC might not be sufficient to justify piercing, but this does not “cleanse” the
inequitable nature of the distribution of proceeds to the three members when ABC was dissolved
while the firm’s debts remained unpaid.
[NOTE: The question specifically asks whether Alice, Bob, and Carla are “personally
liable to the designer and not whether the three might be liable to the LLC as members who
consented to an improper distribution by the LLCthat is, a distribution that rendered the
company unable to pay its debts as they became due. See ULLCA § 406(a) (imposing liability on
members in a member-managed LLC who consent to an improper distribution, as defined in
ULLCA § 405).]
[NOTE: California, Idaho, Iowa, Nebraska, Utah, and Wyoming have enacted the 2006
act. (Kansas and Minnesota are considering doing so.)]
37
CONTRACTS ANALYSIS _____
(Contracts I.A.1.; II.G.)
ANALYSIS
Legal Problems
(1) Is there a contract between the manufacturer and the chef for the purchase and sale of
10 knives?
(2) Is enforceability of the contract between the manufacturer and the chef subject to the
statute of frauds?
(3) Does the document sent by the manufacturer to the chef satisfy the applicable statute
of frauds?
(4) Is the chef’s handwritten note to the manufacturer a “record in confirmation” of the
contract sufficient to satisfy the statute of frauds?
(5) Does the manufacturer’s shipment of 6 knives and the chef’s acceptance of those 6
knives establish the existence of a contract sufficiently to allow its enforcement as to all
10 knives?
DISCUSSION
Summary
The manufacturer and the chef clearly entered into a contract pursuant to which the manufacturer
agreed to sell 10 knives to the chef for $1,000. Because it is a contract for the sale of goods, the
contract is governed by Article 2 of the Uniform Commercial Code. Even though the contract
exists, a contract for the sale of goods for a price of $500 or more is not enforceable against a
party unless there is a writing sufficient to indicate that a contract has been made that is signed
by that party (or unless an exception applies).
Here, the contract price is more than $500, so a writing signed by the manufacturer is
required in order to enforce the contract against it unless an exception applies. The document
included with the knives is a writing sufficient to indicate that a contract has been made, but it is
not clear whether that writing has been “signed” by the manufacturer. If the document constitutes
a writing signed by the manufacturer, the contract is enforceable against the manufacturer for
only six knivesthe quantity mentioned in that writingand not for the remaining four knives.
Even if the document does not constitute a writing signed by the manufacturer, the fact that the
manufacturer delivered, and the chef paid for, six knives would make the contract enforceable
for the six knives but not the remaining four knives.
Point One (20%)
The manufacturer and the chef entered into a contract at the trade show when they agreed that
the chef would buy 10 knives from the manufacturer for $100 each.
38
Contracts Analysis
Because the knives are goods (see UCC § 2-105(1)), the situation presented by these facts is
governed by Article 2 of the Uniform Commercial Code. UCC § 2-204(1) provides that “[a]
contract for sale of goods may be made in any manner sufficient to show agreement, including
conduct by both parties which recognizes the existence of such a contract.” The conversation
between the manufacturer and the chef clearly shows an agreement pursuant to which the
manufacturer would sell, and the chef would buy, 10 knives at $100 each. Thus, UCC § 2-204 is
satisfied and there is a contract for the chef to buy 10 knives from the manufacturer.
Point Two (15%)
The contract between the manufacturer and the chef is a contract for the sale of goods for a price
of $500 or more and therefore is subject to the statute of frauds in Article 2 of the Uniform
Commercial Code.
UCC § 2-201(1) provides that “a contract for the sale of goods for the price of $500 or more is
not enforceable by way of action or defense unless there is some writing sufficient to indicate
that a contract for sale has been made between the parties and signed by the party against whom
enforcement is sought . . . .” Here, the contract between the manufacturer and the chef was for
the sale of goods (the knives) for a price of $1,000. Therefore, it is subject to the writing
requirement of UCC § 2-201(1) unless an exception applies.
Point Three (25%)
The document enclosed in the shipping box with the knives is sufficient to indicate that a
contract has been made, but it is not clear that it has been “signed” by the manufacturer—the
party against whom enforcement is sought. If it is a sufficient writing, it makes the contract
enforceable, but only to the quantity of goods stated in the writing: six knives.
UCC § 2-201(1) requires that, for a contract within its scope to be enforceable, there be a writing
sufficient to indicate that a contract for sale has been made that is signed by the party against
whom enforcement is sought. The facts indicate that the document enclosed with the six knives
that were shipped to the chef stated in its entirety: “It is a pleasure to do business with you.
Enclosed, pursuant to our agreement, are six knives. Please remit the balance due of $600 at your
earliest convenience.” This document is a writing. See UCC § 1-201(b)(43). Further, its contents
clearly indicate that a contract for sale has been made: it states that the knives are enclosed
“pursuant to our agreement” and expresses “pleasure to do business with” the chef.
39
In order for the contract to be enforceable against the manufacturer under UCC
§ 2-201(1), the document must be “signed” by the manufacturer. The facts do not state that the
document bore a signature (in the conventional sense of that term) of the manufacturer, but they
do state that the document was on the manufacturer’s letterhead. As the term “signed” is used in
§ 2-201(1), the meaning is broader than simply bearing a conventional signature. “‘Signed’
includes using any symbol executed or adopted with present intention to adopt or accept a
writing.” UCC § 1-201(39). The Official Comment to UCC § 1-201 states that “in appropriate
cases [the symbol] may be found in a billhead or letterhead.” Accordingly, some cases have held
that a document on company letterhead can constitute a signed writing as long as the requisite
intent is present. See, e.g., Automotive Spares Corp. v. Archer Bearings Co., 382 F. Supp. 513
(N.D. Ill. 1974). Thus, it is possible, but not certain, that the requirement of a signed writing is
satisfied by the document, depending on whether the letterhead was used “with a present
intention to adopt or accept” the document.
Contracts Analysis
Although the writing referred to in UCC § 2-201(1) need not contain detailed information
about the contract, “the contract is not enforceable . . . beyond the quantity of goods shown in
such writing.” Here, the document included by the manufacturer with the knives that were
shipped showed a quantity of only 6 knives. Accordingly, even if the document qualifies as a
writing signed by the manufacturer, the document is not sufficient to make the contract
enforceable beyond 6 knives. Thus, this writing alone does not provide a basis for the chef to
enforce the manufacturer’s obligation to ship the remaining 4 knives pursuant to the oral
agreement for a sale of 10 knives.
[NOTE: Some examinees may mention that the manufacturer’s shipment of only 6 knives
(when 10 knives were ordered) violates the perfect tender rule and that the chef could have
rejected those knives. That is true, but irrelevant. The chef was entitled to accept a less than
perfect tender, and such acceptance will not preclude the chef from enforcing the contract for the
full amount—if the chef can overcome the statute of frauds problem.]
Point Four (20%)
The chef’s note is arguably a letter of confirmation of a contract for the sale of 10 knives. If it is,
then it satisfies the statute of frauds because the manufacturer failed to object to it within 10
days. However, because the note was unsigned, it is unlikely to be given this effect.
The statute of frauds can be satisfied in a contract between merchants if the party seeking
enforcement of the contract has sent the other party a confirmation of the contract that would
itself satisfy the statute of frauds against the enforcing party. In other words, if the chef sends a
written contract confirmation to the manufacturer that would bind the chef, the confirmation will
also bind the manufacturer if the manufacturer does not object to it promptly (i.e., within 10
days). See UCC § 2-201(2).
Here, both the chef and the manufacturer are merchants. The manufacturer deals in goods
of the kind at issue, and the chef’s occupation is such that he would have “knowledge or skills
peculiar to the . . . goods involved in the transaction” (restaurant-quality carving knives). See
UCC § 2-104(1).
However, in all likelihood, no confirmation that would satisfy the statute of frauds was
sent by the chef on these facts. Although the chef sent a note to the manufacturer requesting the
remaining four knives, that note must have been signed in order to satisfy the statute of frauds.
While the note was handwritten and accompanied by a signed check, the note itself was not
“signed” by the chef as that term is defined in UCC § 1-201(39). It did not contain any “symbol
executed or adopted with present intention to authenticate” the writing. A creative examinee
might argue that the chef’s decision to handwrite the note indicates an intent to authenticate by
using handwriting alone, but the UCC definition of “signed” seems clearly to contemplate some
authenticating symbol in addition to the content of the writing itself.
Point Five (20%)
The chef’s acceptance and payment for six knives eliminates the statute-of-frauds problem as to
those six knives, but does not make the contract enforceable as to the remaining four knives.
A contract that does not satisfy the statute of frauds is nonetheless enforceable as to “goods for
which payment has been made and accepted or which have been received and accepted.” See
UCC § 2-201(3)(c). In this case, however, payment has been made for only six knives, and only
six knives have been received and accepted. Thus, this exception would not enable enforcement
of the contract for the remaining four knives but, rather, only for the six knives already delivered.
40
Contracts Analysis
[NOTE: There are other exceptions to the statute of frauds that are not at issue on these
facts, but that might be discussed by some examinees. One of those exceptions applies if the
goods are specially manufactured for the buyer and are not suitable for sale to others. See UCC
§ 2-201(3)(a). Here, there is no indication that the knives are specially manufactured.
Another exception applies if the party against whom enforcement is sought admits in
pleading, testimony, or otherwise in court that a contract was made. See UCC § 2-201(3)(b).
Here, there is no indication of such admission by the manufacturer.
Some examinees may also note that the Uniform Electronic Transactions Act (UETA)
and/or the federal Electronic Signature in Global and National Commerce Act (E-SIGN) allow
the writing requirement of UCC § 2-201 to be satisfied by an electronic communication. This is
true, but it does not change the answer or the analysis, as there is no electronic communication
that would satisfy the requirements of UCC § 2-201.]
41
REAL PROPERTY ANALYSIS ____________ ________
(Real Property III.B.4. & F.; IV.A.1. & C.1.; V.B.1.)
ANALYSIS
Legal Problems
(1)(a) Is there an implied warranty against latent defects in a newly constructed home?
(1)(b) Does an implied warranty against latent defects protect a remote grantee?
(2) Is a buyer who purchases real property from a mortgagor personally liable on the
mortgage debt even if the buyer has not expressly assumed the mortgage obligation?
(3) What are the obligations of a grantor who has conveyed title to real property with a
quitclaim deed?
DISCUSSION
Summary
Although under the common law, there were no implied warranties in the sale of a home, this
caveat emptor approach has been abandoned with respect to builders of new homes who
impliedly warrant that the home is free of latent defects. However, courts are divided as to
whether this implied warranty runs to remote grantees who are not in privity with the home
builder. Some courts require privity; others do not, reasoning that harm to a remote grantee is
foreseeable.
A purchaser of real estate from a mortgagor has no liability on the mortgage obligation
unless he expressly or impliedly assumes that obligation. Here, the man did not expressly assume
the woman’s mortgage obligation, but it is unclear if he impliedly assumed it. If the bank
successfully forecloses on the man’s house, he has no right to recover his loss from the woman
because he took under a quitclaim, not a warranty, deed.
Point One(a) (25%)
Because there is an implied warranty against latent defects in the sale of a new home running
from the builder to the buyer, the builder would be liable to the woman, the original buyer, for
damages caused by the defective concrete.
Under the doctrine of caveat emptor, a home builder makes no implied warranties regarding the
condition of the premises. See generally HERBERT HOVENKAMP & SHELDON F. KURTZ, The Law
of Property 534 (6th ed. 2005). Today, however, American courts have largely rejected the
caveat emptor approach. This rejection is based on the view that a home buyer does not stand on
an “equal footing” with the builder and relies upon the builder’s skill and integrity, as well as the
fact that ordinary home buyers are not in a position to discover latent defects. See, e.g., Petersen
v. Hubschman Construction Co., 389 N.E.2d 1154 (Ill. 1979); Terlinde v. Neely, 271 S.E.2d 768,
769 (S.C. 1980). Although courts have variously described the home builder’s warranty as an
implied warranty against latent defects, an implied warranty of fitness, or an implied warranty of
habitability, it is now recognized in almost all states, and it extends to latent defects such as
42
Real Property Analysis
faulty concrete. See generally Sheed, The Implied Warranty of Habitability, New Implications,
New Applications, 8 REAL ESTATE L.J. 281 (1980).
Although the implied warranty is typically called an implied warranty of habitability, in
most states it is not necessary for the man to prove that the house would have been uninhabitable
but for his repair. It is enough that the defect is major, given its nature and the dollar value of the
problem. In this respect, the content of the implied warranty of habitability for the sale of new
homes closely resembles the Uniform Commercial Code implied warranty of merchantability
that arises upon the sale of goods by a merchant. Indeed, some of the courts that first developed
the warranty of habitability patterned it after the UCC warranty.
Thus, under the law of most states today, the builder would be liable for damages to the
woman, the original buyer, for losses caused by the defective concrete.
Point One(b) (25%)
In many, but not all, jurisdictions the home builder’s warranty against latent defects extends to a
remote grantee like the man.
Not all courts have extended the implied warranty of latent defects to remote grantees. Courts
that apply a privity bar have typically done so on the ground that the warranty is contractual in
nature and thus should only run in favor of parties in privity with each other. See Sensenbrenner
v. Rust, Orling, & Neale, Architects, Inc., 374 S.E.2d 55 (Va. 1988). Other courts, perhaps
influenced by the foreseeability standards of tort law, but also relying on the equal vulnerability
of both initial and remote grantees, have extended the warranty to subsequent purchasers. For
example, in Redarowicz v. Ohlendorf, 441 N.E.2d 324 (Ill. 1982), the Illinois Supreme Court
stated:
The warranty of habitability is a creature of public policy. It is a judicial innovation that
has evolved to protect purchasers of new houses upon discovery of latent defects in their
homes. While the warranty of habitability has roots in the execution of the contract for
sale, we emphasize that it exists independently. Privity of contract is not required. Like
the initial purchaser, the subsequent purchaser has little opportunity to inspect the
construction methods used in building the home. Like the initial purchaser, the
subsequent purchaser is usually not knowledgeable in construction practices and must, to
a substantial degree, rely upon the expertise of the person who built the home. If
construction of a new house is defective, its repair costs should be borne by the
responsible builder-vendor who created the latent defect. The compelling public policies
underlying the implied warranty of habitability should not be frustrated because of the
short intervening ownership of the first purchaser; in these circumstances the implied
warranty of habitability survives a change of hands in the ownership.
441 N.E.2d at 330 (internal citations omitted). A similar approach has been adopted in Iowa. See
Speight v. Walters Development Co., Ltd., 744 N.W.2d 108 (Iowa 2008).
Whether the man, as a remote grantee, is protected by the warranty running from the
builder to the woman depends upon which approach governs.
Point Two (35%)
In most, but not all, jurisdictions the man is not liable on the woman’s mortgage obligation
because he did not expressly assume that mortgage obligation. In some jurisdictions, the man
could be held liable on the woman’s mortgage obligation on the theory that he implicitly
assumed this obligation.
43
Real Property Analysis
Because the woman’s mortgage obligation to the bank was recorded, the man took the home
subject to the mortgage, and the bank’s interest had priority over the man’s. Because the bank
has priority, it can foreclose on the home if the mortgage is in default. However, the recording of
a mortgage does not make the mortgagor’s grantee personally liable for the mortgagor’s
obligation.
The predominant rule in the United States is that if a remote grantee takes subject to a
mortgage which the grantee does not assume, the remote grantee is not personally liable on the
debt. See GRANT S. NELSON & DALE A. WHITMAN, REAL ESTATE FINANCE LAW 273–74 (5th ed.
2007). Of course, that does not mean that the remote grantee will not want to voluntarily make
payments on the mortgage in order to prevent the mortgagee from foreclosing on the debt. But it
does mean that if the mortgagee forecloses on the property, it cannot hold the remote grantee
personally liable on the debt or liable for any deficiency resulting from a mortgage foreclosure.
Following this approach, the man would be personally liable on the woman’s mortgage
obligation if the man assumed (i.e., made a commitment to pay) the mortgage. There is no
evidence of an express assumption agreement here. The deed from the woman to the man made
no mention of the woman’s mortgage, and the facts do not describe any express agreement.
In some jurisdictions, a remote grantee who did not expressly assume a mortgage may be
deemed to have impliedly assumed it where, as here, the remote grantee paid the seller only the
difference between what the house was worth and the outstanding balance on the mortgage
obligation. The rationale for this approach is that “[the grantee’s] retention of the vendor’s
money for the payment of the mortgage imposes upon him the duty of protecting the vendor
against the mortgage debt. This must be so . . . for it would seem to be almost intolerably unjust
to permit him to keep back the . . . [money he would have otherwise paid the vendor to purchase
the property].” Heid v. Vreeland, 30 N.J. Eq. 591, 591 (1879). States that follow this approach
include Connecticut, Illinois, Iowa, Maryland, Michigan, Oklahoma, and South Carolina. See
generally NELSON & WHITMAN, supra, 285.
Here, the man was clearly aware of the mortgage; the mortgage was recorded, and the
man paid the woman only $160,000, considerably less than the house was worth ($360,000).
Moreover, the man began making mortgage payments immediately after buying the house and
continued making payments to the bank for a year.
Thus, in a jurisdiction that follows the implied assumption approach, the man may be
liable to the bank. In a jurisdiction that requires express assumption of a mortgage obligation, the
man would not be liable because there is no evidence that he expressly assumed the mortgage.
[NOTE: If the man is deemed to have impliedly assumed the mortgage, then the woman
is merely a surety and would be liable for any deficiency not paid by the man. See RESTATEMENT
(THIRD) OF SURETYSHIP AND GUARANTY § 1, illustration 29 (1996).]
Point Three (15%)
If the bank successfully forecloses on the man’s home, he cannot recover his loss from the
woman because she gave the man a quitclaim deed and quitclaim deeds contain no warranties of
title on which a buyer can sue.
If a seller conveys land on which there is an unsatisfied encumbrance such as a mortgage, and
that encumbrance has priority against the buyer, the seller is liable to the buyer for any loss borne
by the buyer only if the seller warranted that there was no such encumbrance against the
property. Had the woman conveyed the land to the man by a warranty deed, that deed would
have impliedly included a covenant against encumbrances, and the woman would have been
liable to the man for a breach of this covenant.
44
Real Property Analysis
Here, however, the woman conveyed the home to the man by a quitclaim deed. A
quitclaim deed contains no warranties of title, and the buyer taking under a quitclaim deed has no
claim against the seller for damages resulting from encumbrances against the property having a
priority over the buyer’s interest. Here the bank has priority over the man’s interest because its
mortgage was recorded prior to the quitclaim deed to the man, and thus the man had notice of the
mortgage from the record. The man also had actual notice of the mortgage as evidenced by his
making of mortgage payments to the bank. See WILLIAM B. STOEBUCK & DALE A. WHITMAN,
The Law of Property 905, 90809 (3d ed. 2000). Thus, the man is not entitled to any damages
from the woman for the loss he sustained as a result of the foreclosure.
[NOTE: If the examinee concludes that the man is liable for any deficiency (see Point
Two), the man would have a claim against the woman to recover that deficiency on the ground
that he was her surety to guarantee payment of the deficiency.]
45
DECEDENTS’ ESTATES ANALYSIS
(Decedents’ Estates II.F.2., I.12. & L.; III.A.)
ANALYSIS
Legal Problems
(1)(a) Is Mary barred from taking under John’s will because she and John signed a
prenuptial agreement?
(1)(b) Is Mary barred from taking under John’s will because, at the time of John’s death,
there was a divorce action pending between them?
(1)(c) Under John’s will, is John’s biological, but adopted-out, child entitled to take a
share of John’s estate?
(2) Who should be appointed as the personal representative of John’s estate?
DISCUSSION
Summary
At least three-fourths of John’s estate should be distributed to Mary. A prenuptial agreement
waiving a share of a spouse’s estate does not bar a surviving spouse from taking a bequest that
was voluntarily made after the agreement was signed, and a pending divorce does not revoke a
testamentary disposition in favor of a spouse. In states that have adopted Uniform Probate Code
§ 2-119(c) or a like statute, Son is entitled to one-quarter of John’s estate. In other states, it is
unclear whether one-quarter of John’s estate should be distributed to Son; in these states, the
court will construe John’s intent, and there are arguments for and against the claim that John
intended Son to take under his will. The court will appoint Mary as the personal representative of
John’s estate because a surviving spouse has first priority to serve in this capacity. Son may not
be appointed personal representative of the estate because Son is a minor.
Point One(a) (25%)
A prenuptial agreement waiving rights that a spouse might otherwise assert against the estate of
the other does not preclude a surviving spouse from taking a bequest voluntarily devised to the
surviving spouse by a deceased spouse. Thus, Mary is not precluded from taking a share of
John’s estate by the prenuptial agreement that she signed.
A prenuptial agreement in which spouses waive rights to a share of each other’s assets upon
death or divorce does not bar either party from making subsequent gifts or bequests to the other
spouse. In re Estate of Geyer, 533 A.2d 423, 427 (Pa. 1987); In re Hillegass’ Estate, 244 A.2d
672 (Pa. 1968); L.S. Tellier, Spouse’s Right to Take Under Other Spouse’s Will as Affected by
Antenuptial or Postnuptial Agreement or Property Settlement, 53 A.L.R. 2d 475 (1957). Such an
agreement bars only claims that do not arise from a voluntary gift or bequest. Thus, the
prenuptial agreement signed by John and Mary does not bar Mary from taking a share of John’s
estate under his will.
46
Decedents’ Estates Analysis
Point One(b) (25%)
Although divorce bars a former spouse from taking a bequest under a will executed prior to the
divorce, the filing of a divorce action does not. Thus, Mary is entitled to take under John’s will
notwithstanding the pendency of a divorce action at the time of his death.
A bequest may be revoked by operation of law when a change in circumstance occurs that makes
it unlikely that the testator would have wanted a beneficiary named in the will to take under the
will. All states provide that if, after the execution of a decedent’s will, the decedent is divorced, a
bequest in favor of the decedent’s former spouse is revoked by operation of law. See UNIF.
PROBATE CODE § 2-804. However, the typical statute does not apply if a divorce proceeding is
pending when one of the spouses dies. This approach derives from the fact that, until the divorce
is finalized, no property division order will be entered; thus the moral and legal claims of each
spouse in the property of the other have not yet been satisfied.
Point One(c) (35%)
In most states, Son will be entitled to a share of John’s estate only if the court concludes that
John intended to include an adopted-out child in the bequest to “my children who survive me.”
However, John’s adopted-out child, Son, is entitled to a share of John’s estate under UPC
§ 2-119(c) or a like statute.
Under the terms of John’s will, one-fourth of John’s estate is distributable to John’s “children
who survive me.” Son is a biological child of John who survived John. However, John consented
to Son’s adoption by Aunt, and an adoption typically severs the parent-child relationship
between the child and his biological parents. See UNIF. PROBATE CODE § 2-119(a). A few states
do not sever the parent-child relationship when the child is adopted by a relative of a biological
parent. See, e.g., 755 ILL. COMP STAT. 5/2-4(d)(1); UNIF. PROBATE CODE § 2-119(c) (2008)
(adopted in North Dakota and Utah). In these states, Son would take under John’s will.
In most states, Son would take only if the court concluded that John, when using the
words “my children” in his will, intended to include an adopted-out child. The evidence on this
question is far from determinative. On the one hand, John gave Son up for adoption to a relative,
Aunt, and he took Son into his own home after Aunt’s death. John also told Mary that he was
Son’s father, and he did so after the will was executed. On the other hand, there is no evidence
that John attempted to formalize his relationship with Son. It is thus unclear whether Son would
take a share of John’s estate.
Point Two (15%)
Mary will be appointed personal representative of John’s estate.
An individual named as personal representative in a decedent’s will has priority to receive letters
testamentary from the court overseeing the administration of the estate. See UNIF. PROBATE
CODE § 3-203(a)(1). Where, however, the will is silent regarding the appointment of the personal
representative, the court will appoint a person granted priority under the governing statute if that
person is otherwise qualified. Typically, the decedent’s surviving spouse is the individual with
the first priority. See id. § 3-203(a)(2).
Under the Uniform Probate Code, a surviving spouse has first priority only if the spouse
is a devisee of the decedent. Mary satisfies that condition. Therefore, Mary will be appointed
personal representative of the estate. Even if Mary were not a devisee, she would still be entitled
to be appointed under the Code because John’s devisees (i.e., “my children”) are not qualified to
47
Decedents’ Estates Analysis
be appointed personal representatives because Son is a minor. See UNIF. PROBATE CODE
§ 3-203(a)(4).
48
National Conference of Bar Examiners
302 South Bedford Street | Madison, WI 53703-3622
Phone: 608-280-8550 | Fax: 608-280-8552 | TDD: 608-661-1275
www.ncbex.org
e-mail: contact@ncbex.org